Вы находитесь на странице: 1из 177

IASbaba.

com

S&T, Current Affairs and India Year Book

1. Recently, ISRO has announced to provide navigational support to Indian Railways


through GPS Aided Geo Augmented Navigation (GAGAN) system. Consider the following
regarding GAGAN
1. It is an indigenous navigational guide system on the lines of GPS system jointly
developed by ISRO and DRDO
2. Apart from Communication and navigation, One essential component of the GAGAN
project is the study of the ionospheric behaviour over the Indian region
Select the correct codes
1.
2.
3.
4.

Only 1
Only 2
Both
None

Solution: 2
Explanation:
GAGAN is a joint project of ISRO and Airport Authority of India not DRDO. One essential
component of the GAGAN project is the study of the ionospheric behavior over the Indian
region. This has been specially taken up in view of the rather uncertain nature of the
behaviour of the ionosphere in the region. The study will lead to the optimization of the
algorithms for the ionospheric corrections in the region.
To study the ionospheric behaviour more effectively over entire Indian airspace, Indian
universities and R&D labs, which are involved in the development of regional based
ionotropic model for GAGAN, have suggested nine more TEC stations

Iasbaba.com

Page 1

IASbaba.com

2. Consider the following statements regarding two of the orthodox Schools of Hindu
Philosophy, YOGA and SAMKHYA
1. Samkhya, among the six orthodox schools of Hindu Philosophy is closely related to
Yoga.
2. The first use of the root of word YOGA is found in Rig Veda but its modern meaning
is not relevant to Vedic description.
3. Along with Hinduism Yoga is also practiced in Buddhism, Jainism and Tantra
Philosophy.
Choose the correct code
1.
2.
3.
4.

1 and 2
2 and 3
1, 2 and 3
1 and 3

Solution: 3

Explanation: 21st June is celebrated as International Yoga Day. Hence a question on Yoga is
expected this year.
Samkhya or Sankhya is one of the six orthodox schools of Hindu philosophy. It is described
as the rationalist school of Indian philosophy. It is most related to the Yoga school
of Hinduism, and its rationalism was influential on other schools of Indian philosophies.
he first use of the root of word "yoga" is in hymn 5.81.1 of the Rig Veda, a dedication to
rising Sun-god in the morning (Savitri), where it has been interpreted as "yoke" or "yogically
control".
Rigveda, however, does not describe yoga with the same meaning or context as in modern
times. Early references to practices that later became part of yoga are made in
Brihadaranyaka Upanishad, the earliest Hindu Upanishad.
Yoga is practiced in all the mentioned philosophies.

Iasbaba.com

Page 2

IASbaba.com

3. Ideally, under a global warming scenario the land temperature should increase greatly
in the hot summers and serve as a strong monsoon driver. But in Indian subcontinent the
opposite is happening and it is one of the causes of low rainfall in Monsoon season. What
can be possible reasons for low rainfall in India subcontinent?
1. A strong warming of Indian Ocean that reverses the Land-Sea Thermal Gradient.
2. Increase in land-sea temperature difference due to suppressed warming over the
Indian land mass because of increased aerosol levels.
3. A warming Indian Ocean has resulted in surplus rains over the ocean at the cost of
the monsoon rains over land, simultaneously drying the Indian subcontinent.
Select the correct codes
1.
2.
3.
4.

1 and 2
2 and 3
1 and 3
All

Solution: 3
Explanation:

The land has been cooling and the ocean warming and the monsoon has shown a decreasing
trend during the past century.
The summer monsoon has been showing a weakening trend over the past century with
decreasing rainfall over large regions of the Indian subcontinent. The monsoon occurs
because the land heats up much more than the ocean and the warm air over the land rises
and results in low pressure. This causes the rain-bearing winds from the relatively cooler
ocean to blow on to the land and cause rainfall. That is, it is the strong thermal contrast
between land and ocean that results in a strong monsoon.
Ideally, under a global warming scenario the land temperature should increase greatly in the
hot summers and serve as a strong monsoon driver. But, in the case of the Indian
subcontinent, over the past century, that has not been the case.
Quite a few other studies indicate that the monsoon rainfall is weakening over the South
Asian region during the past half century (since 1950s). Some of these studies suggest that
though the extreme rainfall events have increased over some regions, the frequency of
moderate-to-heavy rainfall events has decreased over the subcontinent.
Iasbaba.com

Page 3

IASbaba.com

The reduction in land-sea temperature contrast is attributed mostly to a strong warming in


the Indian Ocean on a multi-decadal scale with the latest reason being climate change under
a global warming scenario.
The surface warming in the Indian Ocean, especially in the western regions has reached
values of up to 1.2 degrees C during the past century, much larger than the warming trends
in other tropical oceans. The decrease in the land-sea thermal contrast surface temperature
trends (1901-2012) is also visible in the upper atmosphere, as the warming trends in the
ocean surface are transferred to the atmosphere above through convective processes.
Apart from the ocean warming, a part of the decrease in land-sea temperature difference is
also due to suppressed warming over the Indian land mass, possibly due to increased
aerosol levels. Aerosols in the atmosphere reflect the suns heat back into space and cause a
cooling effect.
The warming Indian Ocean also plays a role in weakening the monsoon circulation.
Increased warming in the ocean enhances the large-scale upward motion of warm moist air
over the equatorial ocean. This enhanced upward motion over the ocean is compensated by
subsidence of dry air over the subcontinent, inhibiting convection and rainfall over the
Indian landmass. This means that a warming Indian Ocean has resulted in surplus rains over
the ocean at the cost of the monsoon rains over land, simultaneously drying the Indian
subcontinent.

4. Consider the following statements regarding Rock Cut architecture (caves) of Badami
1. It is the oldest Rock Cut Architecture of India after Barabar Caves of Bihar and Ajanta
Caves of Maharashtra
2. The caves incorporates two faiths i.e Hinduism and Buddhism only
3. Jainism faith is also found in the rock cut caves of Badami
Select the correct codes
1.
2.
3.
4.

1 and 2
Only 2
Only 1
Only 3

Iasbaba.com

Page 4

IASbaba.com

Solution: 4
Explanation:
While India is home to over 1,500 rock-cut works of architecture with the earliest one being
the Barabar caves in Bihar (3rd century BC) and later additions such as Ajanta (2nd century
BC) and Ellora in Maharashtra, the rock-cut shrines of Badami in Bagalkote district of
Karnataka remain as a symbol of Chalukyan architecture (6th to 8th century AD) beginning
from the regime of Pulikesin-I (543-566 AD).
The fourth cave is Jaina which is constructed lastly among all the caves. It is only jain
monument of early chalukya period in badami town and it was made in late 6th-7th century.
Cave 5 is a natural cave of small dimensions with a Buddha statue carved inside.
Rest Caves, 1, 2 and 3 are devoted to Hindu deities.
Source: Friday Review-The Hindu
http://www.thehindu.com/features/friday-review/badami-rockcut-shrines-in-karnatakasbagalkote-district-unique-synthesis-of-styles/article7180698.ece

5. Consider the following statements with respect to LEGUMES


1. Legumes have an ability to fix nitrogen from the atmosphere
2. Legumes help in increasing soil fertility
3. Legumes are highly nutritive but also contain anti-nutrients that are harmful
Select the correct codes
1.
2.
3.
4.

1 and 2
2 and 3
1, 2 and 3
Only 1

Solution: 3
Explanation
The legume family consists of plants that produce a pod with seeds inside. Common edible
legumes include lentils, peas, chickpeas, beans, soybeans and peanuts.

Iasbaba.com

Page 5

IASbaba.com
Cultivation of all the crops including cereals and regular vegetables will get benefitted if the
legume vegetables are grown once a season.

Apart from fixing atmospheric nitrogen, the legume vegetables also help in increasing the
organic content of the soil as they produce huge quantum of foliage that gets added into
the soil. In addition to this, they also enhance the water-holding capacity of soil, he points
out. They have a positive impact on human health too as these protein-rich legume
vegetables can help fight protein and energy-deficiency induced malnutrition.
They also contain so-called anti-nutrients like Phytic Acid, Lectins and Saponins which are
substances that can interfere with digestion and the absorption of other nutrients.

6. Bhama Kalapam, Parvathi Parinayam and Bhakta Prahlada are related to which of
the dance forms
1.
2.
3.
4.

Kathakali
Kuchipudi
Bharatnatyam
Odissi

Solution: 2
Explanation: Taken from Friday Review

http://www.thehindu.com/features/friday-review/guru-vedantam-radhesyam-presentskuchipudi-yakshaganas/article7329312.ece

7. Indian ranks third in World Doping Report published by WADA. Consider the following
statements w.r.t Doping1. Blood doping is the practice of boosting the number of white blood cells in the
bloodstream in order to enhance athletic performance.
2. PerflouroCarbons (PFCs) are chiefly used as Blood substitutes for Blood doping
agents.
3. Gene doping is the therapeutic use of cells, genes, genetic elements, or of the
modulation of gene expression, having the capacity to improve athletic
performance.

Iasbaba.com

Page 6

IASbaba.com

Select the correct codes


1.
2.
3.
4.

1 and 2
Only 1
2 and 3
1, 2 and 3

Solution: 1
Explanation: First two statements are correct. Gene doping is defined by the World AntiDoping Agency as "the non-therapeutic use of cells, genes, genetic elements, or of the
modulation of gene expression, having the capacity to improve athletic performance.

8. Recently Indian Government has given nod for the disinvestment of NTPC and IOC.
Consider the following statements with respect to National Investment Fund created
under disinvestment policy-

1. It is maintained outside Consolidated Fund of India


2. The utilization of funds is done only for the funding of social sector scheme.
3. NIF funds can be utilized for the Recapitalization of public sector banks and
investment in Indian Railways towards capital expenditure
Which is/are the correct codes?
1.
2.
3.
4.

1 and 2
Only 2 and 3
Only 1
1 and 3

Solution: 1 and 3
Explanation: Importance of this question
This question was made considering economy in current affairs. Last week, May 2015,
Government has given nod for 5% disinvestment in NTPC and 10% in IOC. National
Investment Fund that was created under Disinvestment Policy of India got restructured in
2013. Now its utilization is not limited to social sector schemes but also for recapitalization
of PSUs, investment in Indian Railways and RRBs/NABARD etc.
Iasbaba.com

Page 7

IASbaba.com
To know more- http://www.divest.nic.in/Nat_inves_fund.asp

9. Consider the statements with respect to warm blooded and cold blooded animals
1. A cold-blooded animal can convert much more of its food into body mass compared
with a warm-blooded animal.
2. Warm blooded animals require much less energy than cold blooded to survive.
3. Cold blooded animals have stronger immune system than warm blooded due to
capacity to regulate their body temperature
Select the correct codes
1.
2.
3.
4.

Only 1
2 and 3
1 and 3
All

Solution: Only 1
Explanation: Why is this question important?
Recently, Worlds first fully warm-blooded fish named Opah or moonfish was discovered.
UPSC in general never asks primary or secondary issues, like it will never ask the name of
the fish or so? But it dissects the issue at a tertiary level and checks on the broader issue.
This question is framed taking the above into consideration.
Only a small amount of the food that a warm-blooded animal eats is converted into body
mass. The rest is used to fuel a constant body temperature. A cold-blooded animal can
convert much more of its food into body mass compared with a warm-blooded animal.
Being cold-blooded, however, also has its advantages. Cold-blooded animals require much
less energy to survive than warm-blooded animals do. Mammals and birds require much
more food and energy than do cold-blooded animals of the same weight. This is because in
warm-blooded animals, the heat loss from their bodies is proportional to the surface area of
their bodies, while the heat created by their bodies is proportional to their mass.
Another disadvantage to being warm-blooded is that warm-blooded bodies provide a nice
warm environment for viruses, bacteria and parasites to live in. Mammals and birds tend to
have more problems with these infections than do reptiles, whose constantly changing body
temperatures make life more difficult for these invaders. However, an advantage of this is
that mammals and birds have developed a stronger immune system than cold-blooded
animals.
Iasbaba.com

Page 8

IASbaba.com

10. National Green Tribunal has banned the burning of garbage around Delhi-Near Capital
Region recently. Some of the pollutants contained in the smoke from open burning of
garbage can include:
1.
2.
3.
4.
5.
6.

Dioxins
Furans
Arsenic
Mercury
PCBs
Lead

Choose the correct options


1.
2.
3.
4.

1, 2, 3, 4 and 5
1, 2, 3, 5 and 6
1, 3 and 5
All

Solution: All
Explanation: Carbon monoxide, Nitrogen oxides, Sulphur oxides, Hydrochloric acid are also
released but most of you know about these. The most important aspect of this question is
to make you aware about Dioxin. UPSC may be interested in Dioxin. Dioxins are
classified as persistent, bioaccumulative, and toxic pollutants (PBTs). Check previous
years papers and look for UPSCs love towards such questions.
PBTs are highly toxic, long-lasting substances that can build up in the food chain to levels
that are harmful to human and ecosystem health. Persistent means they remain in the
environment for extended periods of time. Bioaccumulative means their concentration
levels increase as they move up the food chain. As a consequence, animals at the top of the
food chain (such as humans) tend to have the highest dioxin concentrations in their bodies.
Dioxins are potent toxicants with the potential to produce a broad spectrum of adverse
effects in humans. Dioxins can alter the fundamental growth and development of cells in
ways that have the potential to lead to many kinds of impacts.

Iasbaba.com

Page 9

IASbaba.com

11. Consider the following statements with respect to Permanent Account Number
(PAN)?
1. Unlike the Aadhaar Number and Driving License, it is issued to foreign nationals.
2. It is accepted as Proof for Indian Citizenship
Choose the correct code/s
1.
2.
3.
4.

Only 1
Only 2
Both
None

Solution: None
Explanation: Recently government has made PAN mandatory for private firms seeking
central excise registration.
Now, focus on the broad issue. A foreign national is entitled to get Aadhar, DL and PAN. So,
first statement is wrong. Also, PAN is not a proof for citizenship rather it is just for financial
transaction.
12. Iodine-125 has significant applications in medicines. Consider the following statements
1. It is used in treatment of cancer
2. It can be used for treating retinoblastoma, a rare form of eye cancer
3. It is produced from nuclear reaction using Xenon
Identify the correct statements
1.
2.
3.
4.

1 and 2
Only 2
1,2 and3
1 and 3

Solution: 3
Explanation: BARC-made BARC I-125 Ocu-Prosta seed is an ideal choice to treat
retinoblastoma and uveal tract melanoma, two forms of rare eye cancers. UPSC has asked
many questions from the achievements of Indian Institutes like ISRO, BARC, TIFR etc. Also,
under development of Science and Technology, this issue is important.

Iasbaba.com

Page 10

IASbaba.com

Iodine-125 is a radioisotope of iodine which has uses in biological assays, nuclear


medicine imaging and in radiation therapy as brachytherapy to treat prostate
cancer and brain tumors. BARC scientists produce Iodine 125 in a batch process by
irradiating 4 gramme of xenon-124 gas in the Dhruva reactor for a period of 15 days. Xenon
-125 produced by the neutron interaction decays into I-125.

13. Aditya-1 is a spacecraft, whose mission is to study the Sun. It will study?
1. Suns corona
2. Suns photosphere
3. Suns chromospheres
Select the correct code
1.
2.
3.
4.

1 only
1 and 3
2 only
All

Solution: 1
https://directory.eoportal.org/web/eoportal/satellite-missions/a/aditya-1

14. With reference to Phyto-chemicals, consider the following


1.
2.
3.
4.

These are chemical compounds, naturally occurring in plants and animals.


They posses antioxidant and hormone like activity.
They can be found in apples, tea and carrot
They have potential to treat cancer, cardiovascular diseases, diabetes etc

Select the correct codes


1.
2.
3.
4.

1,2 and 3
1,2 and 4
2,3 and 4
1,2,3 and 4

Iasbaba.com

Page 11

IASbaba.com
Solution: 3

Explanation: Phyto-chemicals are found only in Plants.


http://pib.nic.in/newsite/efeatures.aspx
http://en.wikipedia.org/wiki/Phytochemical

15. Which of the following international agreement is/are related to Engineering?


1. Washington Accord
2. Sydney Accord
3. Dublin Accord
Select the correct code
1.
2.
3.
4.

1 and 2
2 and 3
Only 1
1, 2 and 3

Solution: 4
http://www.thehindubusinessline.com/news/india-becomes-permanent-member-ofwashington-accord/article6112160.ece
http://www.ieagreements.org/dublin/
http://www.ieagreements.org/sydney/

16. With reference to Light Water Reactor, consider the following


1. Pressurized water reactor and Boiling water reactor is a subtype of LWR
2. The fuel used is of liquid form of Uranium-235
3. Normal water is used as coolant and moderator
Correct statement is/are
1. 1 and 2
Iasbaba.com

Page 12

IASbaba.com
2. Ony 1
3. Only 2
4. 1 and 3

Solution: 4
Explanation: Uranium used is solid in the form of pellets not liquid

17. Bollgard technology is associated with which crop?


1.
2.
3.
4.

Cotton
Wheat
Rice
Pulse

Solution- Cotton

18. Globally, Preterm Birth (PTB) is the single largest cause of neonatal deaths. Consider
the following statements wrt PTB
1. A birth that takes place before the mother has been pregnant for 7 months
construes a preterm birth.
2. India has the highest number of PTB worldwide.
3. Major reason of PTB is the lack of healthcare facilities in the country.
Identify the incorrect statement/s
1.
2.
3.
4.

Only 3
1, 2 and 3
1 and 3
1 and 2

Solution: 3
Explanation:
Globally, Preterm Birth (PTB) is the single largest cause of neonatal deaths. A birth that
takes place before the mother has been pregnant for at least 37 weeks construes a preterm
birth. In India, among the total 27 million babies born annually, 3.6 million babies are born
Iasbaba.com

Page 13

IASbaba.com
preterm, and over 300,000 of these preterm babies die each year because of associated
complications. India, with its highest number of PTBs and the highest number of preterm
deaths worldwide, contributes 25% of the overall global preterm related deaths. The effects
of PTB extend beyond the early infancy with substantial long-term consequences in late
childhood and adult life.

A major reason for our inability to significantly reduce the burden of preterm birth is that
the causes of preterm birth are not fully understood. Preterm birth is likely caused by a
combination of physical, environmental and biological factors. Biological factors appear to
be the predominant causes that predispose a woman to deliver babies preterm.

19. With reference to Indian livestock, what are Gramapriya and Vanaraja?
1.
2.
3.
4.

A breed of Chicken
A breed of Goat
Gramapriya is a breed of Chicken and Vanaraja is a breed of Goat
None

Solution: 1

20. Consider the following Deserts of the world


1.
2.
3.
4.
5.
6.

Namib
Colorado Plateau
Kalahari
Atacama
Great Victoria
Patagonian

Which can be categorized in cold desert?


1.
2.
3.
4.

1,2,3, 5 and 6
2, 3 ,5 and 6
1,2,4,5 and 6
1, 2 ,4 and 6

Solution: 4
The cold Deserts areNamib
Iasbaba.com

Page 14

IASbaba.com
Colorado Plateau
Atacama
Patagonian

21. Consider the following with respect to fundamental forces on Earth


1. The range of Gravitational and Electromagnetic Force are almost equal in nature.
2. Gravitational Force is a strong force as compared to Weak Nuclear force.
Select the correct statement/s
1.
2.
3.
4.

Only 1
Only 2
Both
None

Solution: (1)
Explanation:
Range of Gravitational Force- Infinite
Range of Electromagnetic Force- Infinite
Strength of Gravitational Force- 10^-36
Strength of Weak Nuclear Force- 10^-13

22. Consider the following statements


1. Law of Gravitation at the Moon is One-Sixth that at the Earth.
2. Acceleration due to gravity is the same both on the Moon and the Earth.
Select the correct statement/s
1.
2.
3.
4.

Only 1
Only 2
Both
None

Solution: (4)
Explanation:
Iasbaba.com

Page 15

IASbaba.com
The laws of nature are the same everywhere in the universe. The acceleration due to
gravity at the moon is one-sixth that at the earth, but the law of gravitation is the same
both on the moon and the earth.)

23. It is said that Ancient Indian science had contributed significantly in various aspects of
scientific advancement. Consider the following with respect to the development of
science in Ancient India
1. The difference between Wave and Current was fully understood in Ancient Indian
science.
2. Ancient scientists were able to distinguish between various types of rotational,
translational and vibrational motions.
Select the correct code/s
1.
2.
3.
4.

Only 1
Only 2
Both
None

Solution: (3)
Explanation:
Ideas on Motion in Ancient Indian Science: Ancient Indian thinkers had arrived at an elaborate

system of ideas on motion. Force, the cause of motion, was thought to be of different kinds:
force due to continuous pressure (nodan), as the force of wind on a sailing vessel; impact
(abhighat), as when a potters rod strikes the wheel; persistent tendency (sanskara) to move
in a straight line (vega) or restoration of shape in an elastic body; transmitted force by a
string, rod, etc. The notion of (vega) in the Vaisesika theory of motion perhaps comes
closest to the concept of inertia. Vega, the tendency to move in a straight line, was thought
to be opposed by contact with objects including atmosphere, a parallel to the ideas of
friction and air resistance. It was correctly summarised that the different kinds of motion
(translational, rotational and vibrational) of an extended body arise from only the
translational motion of its constituent particles. A falling leaf in the wind may have
downward motion as a whole (patan) and also rotational and vibrational motion (bhraman,
spandan), but each particle of the leaf at an instant only has a definite (small) displacement.
There was considerable focus in Indian thought on measurement of motion and units of
length and time. It was known that the position of a particle in space can be indicated by
distance measured along three axes. Bhaskara (1150 A.D) had introduced the concept of
instantaneous motion (tatkaliki gati), which anticipated the modern notion of
instantaneous velocity using Differential Calculus. The difference between a wave and a
Iasbaba.com

Page 16

IASbaba.com
current (of water) was clearly understood; a current is a motion of particles of water under
gravity and fluidity while a wave results from the transmission of vibrations of water
particles

24. Amongst the following, the examples of Contact Forces are


1. Electromagnetic Force
2. Gravitational Force
3. Frictional Force
4. Air resistance
5. Viscous Force
Select the correct code
1.
2.
3.
4.

1,3,4 and 5
1, 2 and 3
3, 4 and 5
1, 2, 4 and 5

Solution: (3)
Explanation:
Contact force is the force in which an object comes in contact with another object. Contact
forces are also direct forces. Contact forces are a something that pull on it. Contact forces
are ubiquitous and are responsible for most visible interactions between macroscopic
collections of matter. The most common instances of this include friction, normal force,
and tension
A non-contact force is a force applied to an object by another body that is not in direct
contact with it. The most familiar example of a non-contact force is weight. All four known
fundamental interactions are non-contact forces. Contact forces arise also when solids are
in contact with fluids. For example, for a solid immersed in a fluid, there is an upward
bouyant force equal to the weight of the fluid displaced. The viscous force, air resistance,
etc are also examples of contact forces

Iasbaba.com

Page 17

IASbaba.com

25. Carbohydrates are vital source of energy for any living organism. Consider the
following with respect to various types of energy resources in plants and animals.
1. Glycogens are solely coming from animals, specifically made by the liver and the
muscles, while the starch solely comes from the green plants and staple foods like
potatoes and cassavas.
2. Cellulose occurs exclusively in plants and it is the most abundant organic substance
in plant kingdom.
3. Human store glucose as Starch as well as Gycogens coming from plants and animals
respectively.
Select the correct code/s
1.
2.
3.
4.

1 and 2
1 and 3
Only 3
1, 2 and 3

Solution: (1)
Explanation:
Starch is the main storage polysaccharide of plants. It is the most important dietary source
for human beings. High content of starch is found in cereals, roots, tubers and some
vegetables.
Cellulose occurs exclusively in plants and it is the most abundant organic substance in plant
kingdom. It is a predominant constituent of cell wall of plant cells. The carbohydrates are
stored in animal body as glycogen. It is also known as animal starch.
Glycogen is the body's storage form of starch, though it is technically glucose. To
understand this, you must understand that starch is a plant's storage form of glucose.
However, as humans, we store the same glucose as glycogen. Our body is equipped to
contain excess glucose molecules as glycogen rather than starch.

26. Biodiesel can be produced from among the following1. Jatropa


2. Pongamia Pinnata
Iasbaba.com

Page 18

IASbaba.com
3. Sunflower
4. Rape Seeds

Select the correct code


1.
2.
3.
4.

1 and 2
1, 2 and 4
Only 1
All

Solution: (4)
Explanation:
Pongamia Pinnata is a viable non-toxic alternative to jatropha for making biodiesel. Being a
legume, it fixes nitrogen into the soil and is often used as a windbreak between fields on
farms. Bio Diesel fuel can be produced from oilseed plants such as rape seeds, sunflower,
canola and or JATROPHA CURCAS.

27. Recently India applied for membership of Missile Technology Control Regime.
Consider the following statements with respect to MTCR
1. MTCR is a binding treaty between member nations to limit the proliferation of
missiles and missile technology.
2.

The activities of the MTCR are consistent with the UNs non-proliferation and export
control efforts.

Select the correct code


1.
2.
3.
4.

Only 1
Only 2
Both
None

Solution: (2)
Explanation: MTCR is not a treaty itself. The Missile Technology Control Regime (MTCR) is
an informal and voluntary partnership between 34 countries to prevent
the proliferation of missile and unmanned aerial vehicle technology capable of carrying a
500 kg payload for at least 300 km.

Iasbaba.com

Page 19

IASbaba.com
While there is no formal linkage, the activities of the MTCR are consistent with the UNs
non-proliferation and export control efforts. For example, applying the MTCR Guidelines
and Annex on a national basis helps countries to meet their export control obligations under
UN Security Council Resolution 1540.

28. Consider the following regarding BRAHMOS Missile jointly developed by India and
Russia
1. Its range of 290km is governed by Missile technology Control Regime (MTCR) of which
Russia is a member.
2. Its only disadvantage is its surface to surface variant.
3. It works on the principle of Fire and Forget.
Select the correct code
1.
2.
3.
4.

1 and 2
2 and 3
1 and 3
All

Solution: (3)
Explanation: Brahmos is a supersonic cruise missile.
MTCR controls the proliferation of missile and unmanned aerial vehicle technology capable
of carrying a 500 kg payload for at least 300 km. Since India is not a member of MTCR and
Russia is, therefore the joint development could not cross the limit of 300 Km and hence the
missiles range is 290 km
Brahmos can be launched from Submarines, Ships, Land and Air. So, it can be surface to
surface, Submarine to Air and under development Air to Air. Hence second statement is
wrong.

29. Recently Lead (Pb) was making news due to its contamination in Maggie. Consider the
following statements with respect to Lead
1. High concentration of Pb leads to Dyslexia.

Iasbaba.com

Page 20

IASbaba.com
2. One if its major uses is in the glass of computer and television screens, where it shields
the viewer from radiation.

Select the correct code


1.
2.
3.
4.

Only 1
Only 2
Both
None

Solution: (3)
Explanation:
Lead is a major constituent of the lead-acid battery used extensively in car batteries. It is
used as a coloring element in ceramic glazes, as projectiles, in some candles to threat the
wick. It is the traditional base metal for organ pipes, and it is used as electrodes in the
process of electrolysis. One if its major uses is in the glass of computer and television
screens, where it shields the viewer from radiation. Other uses are in sheeting, cables,
solders, lead crystal glassware, ammunitions, bearings and as weight in sport equipment.
Once taken into the body, lead distributes throughout the body in the blood and is
accumulated in the bones. Depending on the level of exposure, lead can adversely affect
the nervous system, kidney function, immune system, reproductive and developmental
systems and the cardiovascular system. Lead exposure also affects the oxygen carrying
capacity of the blood. The lead effects most commonly encountered in current populations
are neurological effects in children and cardiovascular effects (e.g., high blood pressure and
heart disease) in adults. Infants and young children are especially sensitive to even low
levels of lead, which may contribute to behavioural problems, learning deficits and lowered
IQ.
30. The class of compound known as Biocides are used by Archaeologists in the
preservation and reconstruction of archaeological sites. However, it has variety of other
applications like
1.
2.
3.
4.
5.

Antibiotics
Pesticides
Antifouling Products
Anticancer Agent
Antifungal

Iasbaba.com

Page 21

IASbaba.com
Select the correct codes
1.
2.
3.
4.

1, 2, 3 and 5
2,3,4 and 5
2, 3, and 4
All of the above

Solution: 1
Explanation: A biocide is a chemical substance or microorganism which can deter, render
harmless, or exert a controlling effect on any harmful organism by chemical or biological
means. Biocides are commonly used in medicine, agriculture, forestry, and industry.

A biocide can be:


A pesticide- this includes fungicides, herbicides, insecticides, algicides and rodenticides etc
An antimicrobial- this includes germicides, antibiotics, antiviral, antibacterial, antifungal and
antifouling products etc.

31. Consider the following w.r.t Mangroves Forest


1. The Pichavaram mangroves in Tamil Nadu is India's largest Mangrove Forest
2. Asia has the largest concentration of the worlds mangroves except Middle East
Select the correct code
1.
2.
3.
4.

Only 1
Only 2
Both
None

Solution- 4
ExplanationAccording to Forest Survey Report, 2013, West Bengal has almost half of the mangroves
cover of India. Oman, near Muscat, supports large areas of mangroves, in particular at
Shinas, Qurm Park and Mahout Island. In Arabic, mangrove trees are known as qurm, thus
the mangrove area in Oman is known as Qurm Park.

Iasbaba.com

Page 22

IASbaba.com

32. The term micro-satellite is associated with which of the following?


1. Genetics
2. Space Technology
Select the correct code
1.
2.
3.
4.

Only 1
Only 2
Both
None

Solution- 3
Explanation- http://en.wikipedia.org/wiki/Miniaturized_satellite#Microsatellite
http://en.wikipedia.org/wiki/Microsatellite
33. Global-warming potential (GWP) is a relative measure of how much heat
a greenhouse gas traps in the atmosphere. Consider the statements on which GWP
depends
1. the absorption of infrared radiation by a given species
2. the spectral location of its absorbing wavelengths
3. the atmospheric lifetime of the species
Identify the correct statement/s
1.
2.
3.
4.

1 and 2
Only 3
1 and 3
1,2 and 3

Solution- 4
Explanation- https://en.wikipedia.org/wiki/Global-warming_potential
Iasbaba.com

Page 23

IASbaba.com
http://www.thehindu.com/todays-paper/tp-national/india-needs-clear-policy-on-ozonegases/article6205565.ece

34. With reference to Sankirtana, consider the following


1. Ritual singing, drumming and dancing of Manipur
2. It encompasses an array of arts performed to mark religious occasions and various
stages in the life of the Vaishnav people of Manipur plains.
3. It is practiced at the centre of a temple, where performers narrate the lives and
deeds of Krishna through songs and dance
4. It is inscribed on the Representative List of the UNESCO Intangible Cultural Heritage
of Humanity
Incorrect statement is/are
1.
2.
3.
4.

1 and 2
1, 2 and 3
All
None

Solution- 4
http://www.thehindu.com/todays-paper/tp-national/tp-newdelhi/manipuri-sankirtanainscribed-on-unescos-intangible-heritage-list/article5435312.ece

35. With reference to System of Rice Intensification, consider the following


1. It originated in USA
2. Its a water intensive methodology
3. It is a patented methodology and India has access to it.
Correct statement is/ are
1. 1,2 and 3
2. 2 and 3
3. Only 1
Iasbaba.com

Page 24

IASbaba.com
4. None
Solution- 4
Explanation- Originated in Medagascar not USA. It helps in saving water. Not a patent
http://www.thehindu.com/opinion/op-ed/more-rice-from-less-water/article6183223.ece

36. As per Indian culture, the word Kolam is associated with


1. Art of Rangoli in southern parts of the country
2. A type of tribe
3. Art of Weaving of people of ancient origin
Correct statement(s)
1.
2.
3.
4.

Only 1
1 and 2
Only 3
None

Solution- 2
http://www.thehindu.com/news/national/telangana/kolam-women-dish-outknowledge-on-traditional-food/article6318693.ece

37.Albedo is the percentage of incoming radiation reflected off a surface. Consider the
following statements
1. The more radiation reflected the more Global warming that occurs.
2. Polar ice has highest albedo.
Incorrect statement(s)
1.
2.
3.
4.

Only 1
Only 2
Both
None

Solution- 1
Iasbaba.com

Page 25

IASbaba.com
http://www.ecocem.ie/environmental,albedo.htm
http://www.eoearth.org/view/article/149954/

38. Tropical cyclones occur in


1.
2.
3.
4.

tropics only
tropics and sub tropics only
subtropics only
tropic, subtropics and extra-tropics

Solution- 4
In addition to tropical cyclones, there are two other classes of cyclones within the spectrum
of cyclone types. These kinds of cyclones, known as extratropical cyclones and subtropical
cyclones, can be stages a tropical cyclone passes through during its formation or
dissipation. An extratropical cyclone is a storm that derives energy from horizontal
temperature differences, which are typical in higher latitudes. A tropical cyclone can
become extratropical as it moves toward higher latitudes if its energy source changes from
heat released by condensation to differences in temperature between air masses; although
not as frequently, an extratropical cyclone can transform into a subtropical storm, and from
there into a tropical cyclone. From space, extratropical storms have a characteristic
"comma-shaped" cloud pattern. Extratropical cyclones can also be dangerous when their
low-pressure centres cause powerful winds and high seas

39. When water droplets freeze on ice crystals, process known as


1.
2.
3.
4.

Bergeron process
Snowflakes process
Ice Shelves process
Water crystallization

Solution- 1

Iasbaba.com

Page 26

IASbaba.com

40. Which of followings are rift valley1.


2.
3.
4.

Lake Tanganayika
Gulf of Aden
Lake Baikal
Black Sea

Select the correct code


1.
2.
3.
4.

2 and 3
1, 3 and 4
2 and 4
1 and 3

Solution- 4
Many of the world's largest lakes are located in rift valleys. Lake Baikal in Siberia, a World
Heritage Site, lies in an active rift valley. Baikal is both the deepest lake in the world and,
with 20% of all of the liquid freshwater on earth, has the greatest volume. Lake Tanganyika,
second by both measures, is in the Albertine Rift, the westernmost arm of the active East
African Rift. Lake Superior in North America, the largest freshwater lake by area, lies in the
ancient and dormant Midcontinent Rift. The largest subglacial lake, Lake Vostok, may also lie
in an ancient rift valley.

41. Below are the statements that differentiate the types of diabetes. If you notice your
friend is a diabetic, then how will you differentiate if he/she is type 1 or type 2?
1. Your friend is having the most common type of diabetes.
2. His/Her familiar background shows strong hereditary reasons for being diabetic.
Select the correct code
1. Your friend is suffering from Type 2 diabetes
Iasbaba.com

Page 27

IASbaba.com
2. Your friends is suffering from Type 1 diabetes
3. Both
4. None
Solution- 1
Both are the symptoms of Type 2 diabetes. Type 2 is the most common form of diabetes
mainly found in adults and constituting 90-95% as compared to type 1. Genetically both
type 1 and type 2 can be found but type 2 has strong genetic correlation than type 1

42. Arrange the following countries from West to East (Horizontally) along Mediterranean
Sea
1.
2.
3.
4.

Tunisia
Egypt
Jordan
Syria

Select the correct code


1.
2.
3.
4.

1-2-3-4
2-4-3-1
3-4-2-1
1-2-4-3

Solution- 1
Check the Map

Iasbaba.com

Page 28

IASbaba.com

43. A geographical indication (GI) is a name or sign used on certain products which
corresponds to a specific geographical location or origin. Match the following Indian GI
correctly
1.
2.
3.
4.

Bidriware- Kerala
Chanderi Fabric- Madhya Pradesh
Nirmal Paintings- Telangana
Kaipad Rice- Karnataka

Select the correct code


1.
2.
3.
4.

1, 2, 3 and 4
1, 3 and 4
2 and 3
1 and 4

Solution- 3
https://en.wikipedia.org/wiki/List_of_Geographical_Indications_in_India
Iasbaba.com

Page 29

IASbaba.com

44. Consider the following regarding Uranium Enrichment


1. Low Enriched Uranium (LEU) has lower than 30% concentration of Uranium-235 that is a
weapons grade.
2. Highly enriched Uranium (HEU) has more than 30% concentration of Uranium-235 and used
in nuclear reactors and research.
3. Uranium in natural condition is found as U-235 but unless processed through enrichment,
cannot be used for civilian or weapons-grade technologies.
Select the correct statement/s
1.
2.
3.
4.

1 and 2
2 and 3
1, 2 and 3
None

Solution- 4
Natural Uranium is 99 % U-238 isotope. LEU has lower than 20% concentration of U-235- used in
civilian purpose/nuclear reactors. HEU more than 20 % of U-235, used for weapons-grade.

45. Amongst the following that is/are not a STAR1.


2.
3.
4.

Pulsar
Supernova
Quasars
Pluto

Select the correct code


1.
2.
3.
4.

1, 2 and 3
2, 3 and 4
1, 3 and 4
All

Solution- 2
Only Pulsar is a star.

Iasbaba.com

Page 30

IASbaba.com
45. Recently NASA has solved the mystery of formation of Quasars. Consider the following
statements in its reference
1. Quasars are the brightest object in the universe.
2. They exhibit similar properties as galaxies.
Select the wrong statement/s
1.
2.
3.
4.

Only 1
Only 2
Both
None

Solution- 4

http://www.thehindu.com/sci-tech/science/nasas-hubble-finds-formative-years-ofquasars/article7336606.ece

46. Recently RBI announced Strategic Debt Restructuring (SDR) Scheme. It is1. To allow flexible norms for lenders to pay debt.
2. To allow lenders only Public and Private Banks to convert their loans into equity
stake
3. To allow banks and non-banking lending institutions to convert their loans into
equity stake
Select the correct code
1.
2.
3.
4.

1 and 3
2 and 3
Only 2
Only 3

Solution- 4

Iasbaba.com

Page 31

IASbaba.com
The banking regulator RBI issued new norms for Strategic Debt Conversion (SDR) which will
give lenders the right to convert their outstanding loans into a majority equity stake if the
borrower fails to meet conditions stipulated under the restructuring package.
http://www.indiafilings.com/learn/rbi-strategic-debt-restructuring-scheme/

47. Indian Banks are witnessing rising NPAs (Non-Performing Assets) due to the
slowdown in the Indian economy and high interest costs. What are the tools issued by
RBI to tackle rising NPA?
1.
2.
3.
4.

Asset Reconstruction Companies


SARFESI Act
Joint Lenders Forum
KYC norm

Select the correct code1.


2.
3.
4.

1, 2 and 4
2, 3 and 4
1 and 4
All

Solution- 4
All are steps towards controlling NPA.
http://www.indiafilings.com/learn/rbi-strategic-debt-restructuring-scheme/

48. Consider the following statements with respect to Solar Wind and Earth Wind1. The solar wind is very strong compared to the wind on Earth
2. The solar wind is very weak compared to the wind on Earth, though it is much, much
faster than Earth wind.
Select the correct code1. Only 1
2. Only 2
3. Both
Iasbaba.com

Page 32

IASbaba.com
4. None
Solution- 2
Explanation- The solar wind is very weak compared to the wind on Earth, though it is much,
much faster. When we measure solar wind speeds, we typically get speeds of 1-2 million
miles per hour. They end up being weaker because there is very little of it. Solar wind
density is usually about 100 particles per cubic inch. Thus, a typical pressure from the solar
wind is measure in nanopascals whereas at the Earths surface, the atmospheric pressure is
100 kilopascals, and surface winds are about 100 pascals. Since solar wind is measured in
nanopascals it is approximately 1000 million times weaker than winds here on Earth.

49. Recently Solar Storms were in news. Consider the statements with respect to Solar
Storms1. They are very harmful for Humans and other living organisms on Earth.
2. They can affect communication technologies like GPS and radio communications.
3. They can also affect Power Grids and Electrical units.

Select the wrong statement/s


1.
2.
3.
4.

1 and 2
Only 1
Only 2
Only 3

Solution- 1
Explanation- Solar storms are not dangerous to humans on Earths surface. These storms
are awesome to contemplate, but they cannot harm our human bodies as long as we remain
on the surface of Earth, where were protected by Earths blanket of atmosphere.
Remember, theres every reason to believe that storms on the sun have been happening for
billions of years, since the sun and Earth came to be. If thats so, then all life on Earth
evolved under their influence.

Iasbaba.com

Page 33

IASbaba.com
What is the danger of a solar storm in space? Very high-energy particles, such as those
carried by CMEs, can cause radiation poisoning to humans and other mammals. They would
be dangerous to unshielded astronauts, say, astronauts travelling to the moon. Large doses
could be fatal.
Still, solar storms and their effects are no problem for us on Earths surface. Earths
atmosphere and magnetosphere protect our human bodies from the effects of solar flares.
On the other hand solar storms can be dangerous to our technologies. When a coronal mass
ejection, or CME, strikes Earths atmosphere, it causes a temporary disturbance of the
Earths magnetic field. The storm on the sun causes a type of storm on the Earth, known as
a geomagnetic storm. They have the potential to affect power grids, and have been known
to black out entire cities, even entire regions.

50. Amongst the following that can be put in the category of Social insects?
1.
2.
3.
4.

Honeybee
Wasp
Termites
Bumblebees

Select the correct code


1.
2.
3.
4.

1, 2 and 3
1, 3 and 4
1, 2, 3 and 4
1, 2 and 4

Solution- 4
Explanation- All are social insects. Bumblebees were recently in news. India is home to 48 of
the 250 known species of bumblebees. It was revealed by the book Indian Bumblebees
released in April 2015.

51. Consider the following statements with reference to Tornados and Hurricane
1. Tornados are more destructive than Hurricanes under the same circumstance
Iasbaba.com

Page 34

IASbaba.com
2. Both rotate clockwise in Southern Hemisphere and counter-clock wise in northern
Hemisphere.
3. While Hurricanes occurs mostly in warm areas Tornados occurs at places where cold
and warm fronts converges.
Select the correct code
1.
2.
3.
4.

1 and 2
2 and 3
1 and 3
Only 3

Solution- 2
Explanation- First statement is wrong. Although Tornados are more frequent than
Hurricanes but cause less damage as compared to Hurricane. The simple reason is the long
lasting nature of Hurricane- few days to weeks. While Tornados lasts for minutes to hours.
Rest statements are true.
Read here- http://www.diffen.com/difference/Hurricane_vs_Tornado
http://abcnews.go.com/Technology/Weather/story?id=99457

52. Which of the following can be the easiest way to predict the upcoming Earthquake?
1.
2.
3.
4.

Huge release of Methane gas


Release of Radon gas
Release of carbon dioxide
None

Solution- 2
Explanation- Uranium is found in low-levels in all rocks and soil. Radon is a gaseous
radioactive decay product of uranium. As the uranium undergoes radioactive decay,
radioactive radon is generated andtrapped in the rocks that contain the uranium.
The earthquake theory involving radon suggests that prior to the actual quake, there is
some subterranean movement where rocks are crushed, soil is un-compacted and the
trapped radon is released producing a pre-quake spike in radon concentration.

Iasbaba.com

Page 35

IASbaba.com
So radon presents the following attributes:

it is present in all rocks and soil


if a rock is broken or if soil is disturbed, radon will be released
it is gaseous and air currents / thermal gradients will carry it up to the earth's surface
producing a detectable plume
it is radioactive, this makes detecting small amounts or small changes in radon
concentration relatively routine due to well-developed and very sensitive methods
for detecting and accurately measuring radioactivity.
Radon has a very short radioactive half-life, a bit under 4 days. Being a gas and
having a short half-life is very useful in terms of measuring radon emissions. If a
radon emission spike occurs, the gas will dissipate quickly and after about 10 halflives (40 days) normal background levels of radioactivity will return

53. Consider the following bills passed by Parliament in recent times


1.
2.
3.
4.

Finance Bill 2015


National Judicial Appointments Commission Bill, 2014
Insurance laws (Amendment) bill, 2015
Motor Vehicles Amendment Bill, 2015

Select the bills passed through Voice Vote?


1.
2.
3.
4.

2 and 4
1 and 4
1, 2, 3 and 4
None

Solution- 3
Explanation

A voice vote is used in Lok Sabha, Rajya Sabha and state assemblies to vote for
certain resolutions. It is used when there is a wide agreement on issues and in some
cases where the house is not in order. It was used during the formation of Telangana
state, in forming the 29th state of India.
On 14 August 2014, Rajya Sabha passed National Judicial Appointments Commission
Bill, 2014 by a voice vote.

Iasbaba.com

Page 36

IASbaba.com

On 12 November 2014, BJP won majority in Maharshtra vidhan Sabha using voice
vote which was questioned by opposition parties like Shivsena and INC.
On 12 March 2015, the parliament passed the Insurance laws (Amendment) bill,
2015. Earlier it was passed in Lok Sabha on4th March 2015 and then passed by the
Rajya Sabha on 12 March 2015 by voice note. It proposes to increase FDI limit in
local insurers from earlier 26% to 49% now.
On 30 April, 2015, the Lok Sabha passed the Finance Bill for 2015 by a voice vote.
The Lok Sabha on 3 March 2015 passed the Motor Vehicles Amendment Bill, 2015 by
voice vote. The Bill seeks to amend the Motor Vehicles Act, 1988 and replace the
ordinance promulgated in this regard. Earlier, bill was not able to pass in the winter
session of Parliament 2014, so the ordinance was promulgated in January 2015 .

54. Suppose you hold


statement/s

license in Mining sector in India. Consider the following

1. Mining leases can be granted to you up to 50 years except for atomic minerals.
2. Seeing your productive performance, Central government may permit you to acquire
one or more licences or leases covering additional area.
Select the correct code1.
2.
3.
4.

Only 1
Only 2
Both
None

Solution- 4
Explanation- Recently, Mines and Minerals (Development and Regulation) Act, 2015 was passed.
So a tricky questions to check your concepts

Read this- Maximum area for mining: Under the Act, a person could acquire one mining
lease for a maximum area of 10 sq km. However, for the development of any mineral, the
central government could permit the person to acquire one or more licenses or leases
covering additional area. The Bill amends this provision to allow the central government to
increase the area limits for mining, instead of providing additional leases.
Lease period: Under the Act, a mining lease was granted for a maximum of 30 years and a
minimum of 20 years and could be renewed for a period not exceeding 20 years. Under the
Bill, the lease period for coal and lignite remains unchanged. For all minerals other than
coal, lignite and atomic minerals, mining leases shall be granted for a period of 50 years.
All mining leases granted for such minerals before the Bill, shall be valid for 50 years. On
Iasbaba.com

Page 37

IASbaba.com
expiry of the lease, instead of being renewed, the leases shall be put up for auction, as
specified in the Act.

55. Which among the following are called Notified Minerals?


1.
2.
3.
4.
5.
6.

Bauxite
Iron ore
Coal
Uranium
Limestone
Manganese ore

Select the correct ones


1.
2.
3.
4.

2, 3, 4, 5 and 6
1, 2, 5 and 6
2, 3, 4 and 5
3, 4, 5 and 6

Solution- 2

Explanation- Mines and Minerals (Development and Regulation) Bill adds a new Fourth
Schedule to the Act. It includes bauxite, iron ore, limestone and manganese ore and are
defined as notified minerals. The central government may, by notification, amend this
Schedule.

56. Consider the following statements w.r.t Enforcement Directorate


1. It looks after investigation cases related to Foreign Exchange Management Act, 1999
and Prevention of Money Laundering Act, PMLA
2. It falls under Ministry of Finance, Department of Expenditure
3. ED can arrest the offenders in relation to PMLA offence
Select the correct code
1.
2.
3.
4.

1 and 2
2 and 3
1 and 3
All

Solution- 3

Iasbaba.com

Page 38

IASbaba.com
Explanation- ED is in news for quite a long time. Prepare about it. It is part of Department of
Revenue not Expenditure.
57. Which of the following treaties are related to Nuclear Weapons?
1.
2.
3.
4.
5.

Antarctica Treaty
Seabed Treaty
START
NPT
SORT

Select the correct code


1.
2.
3.
4.

1, 4, 5
3, 4 and 5
1, 2,3,4 and 5
2, 3 and 4

Solution- 3
The Strategic Arms Reduction Treaty (START) was signed between the US and Soviet Union
in July 1991. In summary, START I limited the number of heavy bombers (the large aircraft
able to carry heavier bombs including nuclear ones), intercontinental ballistic missiles
(ICBMs), and submarine launched ballistic missiles (SLBMs), to 1,600 in total. These delivery
systems were then limited to deploying no more than 6,000 nuclear warheads in all.

The Strategic Offensive Reductions Treaty (SORT) or Moscow Treaty entered into force in
June 2003 to limit the numbers of operationally deployed nuclear warheads by the US and
Russia to 1,700-2,200 apiece.
Antarctica Treaty
In June 1961 the Antarctica Treaty established the use of this continent as only for peaceful
purposes and it must be free from nuclear weapons deployment and testing.
Seabed Treaty
The Seabed Treaty entered into force in May 1972 with the purpose of forbidding the sitting
of nuclear weapons and other weapons of mass destruction on the seabed or ocean floor
over 12 miles away from any coastal zone.

Iasbaba.com

Page 39

IASbaba.com

58. Which of the following should be your first choice for the treatment of Hyperacidity?
1.
2.
3.
4.

Antacids
Antihistamines
Histamines
Analgesics

Solution- 2
Histamines produce hyperacidity. So, antihistamines should be used. NCERT

59. If you visit a water treatment plant in your area, you will witness the processes
followed in water purification. Select the correct order in which water purification works1.
2.
3.
4.

Flocculation
Coagulation
Sedimentation
Disinfection

Correct order1.
2.
3.
4.

1-2-3-4
2-1-3-4
2-3-4-1
1-3-4-2

Solution- 2
Coagulation-Flocculation-Sedimentation-Disinfection- Read the basics here
https://en.wikipedia.org/wiki/Water_purification#Coagulation_and_flocculation

60. Our food contains number of additives like sweeteners, antioxidants, flavours, fat
emulsifiers and preservatives. Which of the following also add nutritive value to our food?
1. Antioxidants
Iasbaba.com

Page 40

IASbaba.com
2. Preservatives
3. Sweeteners
4. Fat emulsifiers
Select the correct code1.
2.
3.
4.

1, 3, and 4
1, 2, 3 and 4
2, 3, and 4
None

Solution- 4
Explanation- They do not have nutritive values.
61. Artificial sweeteners like Aspartame or Saccharines are 100 times sweeter than Sugar
despite it is recommended for a Diabetic person, why?
1. Artificial sweeteners have less effect on insulin secretion
2. Artificial sweeteners do not add to calorie intake
3. Artificial sweeteners easily digested and excreted while sugar is energy rich and
affect digestive system
Select the correct code1.
2.
3.
4.

Only 1
2 and 3
Only 2
All

Solution- 3
Explanation- Diabetic people need to control their calorie intake and hence artificial
sweeteners like Aspartame or Saccharine is vital for them.

62. Consider the following statements regarding Vitamins


1. If you are eating eggs regularly, the concentration of Vitamin B-2 and Vitamin D is
increasing.
2. Intake of excess green leafy vegetables will enhance the concentration of Vitamin C,
K and B-1
Iasbaba.com

Page 41

IASbaba.com
Select the correct code
1.
2.
3.
4.

Only 1
Only 2
Both
None

Solution- 3
Explanation- Direct, NCERT, prepare about it from the table given in NCERTs
63. Which feature of some species of blue-green algae helps to promote then as biofertilizers?
1. They induce the roots of the crop plants to absorb the soil nitrates in larger
quantities
2. They have the mechanism to convert atmospheric nitrogen into a form that crop
plants can absorb readily.
3. They convert atmospheric methane into ammonia which the crop plants can absorb
readily
4. They induce the crop plants to produce enzymes which help convert atmospheric
nitrogen to nitrates.
Solution- 2
Cyanobacteria, also known as blue-green algae are a phylum of bacteria that obtain their
energy through photosynthesis. They are significant component of the marine nitrogen
cycle and an important primary producer in many areas of the ocean. During crop growth
cycle, the blue-green algae grow, multiply, fix atmospheric nitrogen and make it available to
the crop by way of excretion and autolysis.

64. Excessive release of the pollutant carbon monoxide (CO) into the sir may produce a
condition in which oxygen supply in the human body decreases. Why?
1.
2.
3.
4.

The inhaled CO has much higher affinity for haemoglobin as compared to oxygen
The inhaled CO may be converted into CO2 that causes poisoning
The inhaled CO destroys the chemical structure of haemoglobin
The inhaled CO primarily affects the neurological and respiratory system in the brain.

Solution- 1
Explanation- Haemoglobin contains long polymer like structure where metal carbonyls are
bonded in cage like structure. The metals in the carbonyls are iron and cobalt. Carbon
monoxide has the physical property to replace these metals from metal carbonyls. If CO
Iasbaba.com

Page 42

IASbaba.com
enters our body the first thing they will do is to replace Fe or Co and form bond with
carbonyls. This is serious as Carbon-monoxide is poisonous. Oxygen cannot replace the
metals in haemoglobin but carbon monoxide can easily do so.
65. Ice is less dense than water is a well know fact. What if it would have been denser
than water?
1. Lakes and oceans would freeze from the bottom up, almost certainly preventing the
kind of chemistry that makes life possible.
2. Increasing pressure would raise the freezing point of water. The deep oceans would
be permanently frozen.
3. A very large part of the planets Oxygen is produced by plankton in the ocean and this
could be severely disrupted.
Select the correct code1.
2.
3.
4.

1 and 2
2 and 3
1, 2 and 3
Only 2

Solution- 3
Explanation- All of the above is correct. When the density of ice will be more (hypothetical)
above changes are obvious. With more dense ice, it will not float above the water and
bottom up freezing will happen. With more pressure deep down the ocean and further
increase due to depth, will make deep oceans to freeze forever. The dense ice will not allow
diffusion to take place and hence availability of oxygen will be less, as it is a gas and need
gaseous media to diffuse. Hence, planktons life will be at threat.

66. What are the components of Photochemical Smog?


1.
2.
3.
4.

Nitric oxide, Ozone and PAN (peroxyacetyl nitrate)


Nitric oxide, Ozone, aldehyde and PAN (peroxyacetyl nitrate)
Nitric oxide, Ozone, aldehyde, acrolein and PAN (peroxyacetyl nitrate)
Nitric oxide, Ozone, formaldehyde, acrolein and PAN (peroxyacetyl nitrate)

Solution- 4
Explanation- Direct Question-NCERT class 11th
67. Which among the following can be used as bleaching agents?
Iasbaba.com

Page 43

IASbaba.com
1.
2.
3.
4.

Mix of liquefied carbon dioxide and a detergent


Chloroethene
Hydrogen peroxide
Sodium hypochlorite

Select the correct code1.


2.
3.
4.

1, 2, 3 and 4
2, 3 and 4
1, 3 and 4
3 and 4

Solution- 1
Explanation- All of them can be used. But now adays chloroethene is not used because of
contamination of chloro compounds. Sodium hypochlorite is the most common bleaching
agent.

68. A GM or transgenic crop is a plant that has a novel combination of genetic material obtained
through the use of modern biotechnology. Consider the following regarding GM crops
1. India has the third largest area planted under genetically modified (GM) crops after US and Brazil.
2. The first generation of GM crops resulted into increased nutritional and/or industrial traits.
3. The second generation GM crops will feature increased nutritional and/or industrial traits like Rice
enriched with iron, vitamin A and E, and lysine and Potatoes with higher starch content, and inulin

Select the correct code


1.
2.
3.
4.

1 and 2
2 and 3
Only 1
Only 3

Solution- 4
India has the fourth largest area planted under genetically modified (GM) crops, according to the
International Service for the Acquisition of Agri-Biotech Applications (ISAAA)
In the developed world, there is clear evidence that the use of GM crops has resulted in significant
benefits. These include:
Iasbaba.com

Page 44

IASbaba.com
1.
2.
3.
4.

Higher crop yields


Reduced farm costs
Increased farm profit
Improvement in health and the environment

These first generation crops have proven their ability to lower farm-level production costs.

Now, research is focused on second-generation GM crops that will feature increased nutritional
and/or industrial traits. These crops will have more direct benefits to consumers. Examples include:

1.
2.
3.
4.
5.
6.

Rice enriched with iron, vitamin A and E, and lysine


Potatoes with higher starch content, and inulin
Edible vaccines in maize, banana and potatoes
Maize varieties with low phytic acid and increased essential amino acids
Healthier oils from soybean and canola
Allergen-free nuts

69. Recently India has successfully launched a Canister-Based nuclear capable AGNI-V missile.
Consider the following regarding this
1. An advantage of canister based system is, it can be used for the launch of any missile irrespective
of its size.
2. AGNI-V is the first missile to be tested on Canister based launch system.

Select the correct code1.


2.
3.
4.

Only 1
Only 2
Both
None

Solution- 4

A canister for a missile serves a duel function which provides an environmental container and a
launching tube. A canister launch system can be either hot launch, where the missile ignites in the
cell, or cold launch, where the missile is expelled by gas produced by a gas generator which is not
part of the missile itself, and then the missile ignites. In case of hot launches the problem is the heat
Iasbaba.com

Page 45

IASbaba.com
produced by the missile at the time of launch. The hot launch is better for small missiles as the
ejection part itself will be done by using missiles own engine.
So it depends on the size of missile the feasibility of the system utilized for the ejection.
The Shaurya missile is a canister launched hypersonic surface-to-surface tactical missile developed
by the Indian Defence Research and Development Organisation (DRDO) for use by the Indian Armed
Forces.

70. Consider the following statements with reference to Multiple Independently targetable Reentry Vehincle (MIRV)
1. India possesses MIRV technology along with Russia, USA China, France and Brazil.
2. It is a ballistic missile payload containing several warheads, each capable of being aimed to hit one
of a group of targets.

Select the correct code1.


2.
3.
4.

Only 1
Only 2
Both
None

Solution- 2
India is working on the development of MIRV technology. But right now there is no fully tested
version of MIRV in any of Indian missiles. Only France, China, US, Russia and Brazil possesses this
technology.

71. To bring prosperity and tranquility in north-eastern region of India what actions or plans you
think is/are good from given below
1. Shift from voluminous crops to high value horticulture crops.
2. More strict laws to counter insurgency.
3. Policy to give boost to bamboo based handicraft industries.
4. Laying down of rail tracks to increase connectivity among north-eastern cities.
Select the correct code
Iasbaba.com

Page 46

IASbaba.com
1.
2.
3.
4.

1, 2, 3 only
1, 3 only
All of the above
None

SOLUTION: 2
North-east region has rough terrain so possibility of laying down tracks wouldnt be nice idea to
execute. It has been established by facts in history that peace was brought by negotiation rather
strict laws. Strict laws have yielded an atmosphere of animosity.
72. What are the important commercial crops produced in Malabar region from given below:
1. Coffee and tea
2. Tobacco
3. Betelnut
4. Rubber
5. Cashewnut
Select the correct code
1.
2.
3.
4.

1, 2, 3 only
1, 3. 5 only
All of the above
1, 3, 4, 5 only

SOLUTION: 4
India year book page no. 6 Tobacco is mainly produced in Guntur and Prakasham district of Andhra
Pradesh and Kheda district of Gujrat.

73. According to world biogeographic classification. India represents which two major realms
1. Palearctic and Indo-Malayan
2. Palearctic and Neo-arctic
3. Neo-arctic and Indo-Malayan
Iasbaba.com

Page 47

IASbaba.com
Select the correct code
1.
2.
3.
4.

Only 1
1 and 2
Only 3
None

SOLUTION: 1
India year book page no. 6

74. Consider the statements below:


1. Central tribal belt is dominated by Proto-Australoid racial strain.
2. Hills of southern India is inhabited by tribal of Negritoes racial strain.
3. Negritoes racial strain is confined to islands of Andaman and Nicobar only.
Select the correct code
1.
2.
3.
4.

1, 2 only
1, 3 only
3 only
None

SOLUTION: 1
Read about puliyans, uralis living in southern hills . They are of negritoes racial strain

75. National symbols hold a revered position among Indians. It is/was maintained and protected
by
1. Non-statutory instructions
2. Statutes
3. Constitutional references
Select the correct code
1. 1, 2 only
2. 2, 3 only
3. 3 only
Iasbaba.com

Page 48

IASbaba.com
4. All of the above
SOLUTION: 4
Fundamental duties say about paying respect to national symbols. There are various legislation
pieces like the flag code of India 2002 which were legislated to seek compliance to standards from
citizens.
India year book chapter no. 2

76. Give It Up campaign will have positive impact on


1.
2.
3.
4.

Health of rural India.


Environment.
Revenue of Local bodies.
Comforts of rural women.

Select the correct code


1.
2.
3.
4.

1 only
1, 3 only
1, 2, 4 only
All of the above

SOLUTION: 3

77. Major initiatives envisaged to make Digital India campaign a success include
1.
2.
3.
4.
5.

Communication infrastructure and services


Products like digital lockers, e-sign and e-hospital/ORS etc.
Portals and apps
Opening of research institutions
Important Policies initiatives.

Select the correct code


1. 1, 2. 3only
2. 1, 2. 3. 5 only
3. 3 only
Iasbaba.com

Page 49

IASbaba.com
4. All of the above
Solution: 4

78. Union cabinet proposed amendment to the Prevention of corruption act 1988. Proposed
amendments include
1. Punishment for both bribe giver and bribe taker.
2. Enhances punishment terms.
3. Non-monetary gratification is covered in the definition of gratification in the PCA 1988.
Select the correct code
1.
2.
3.
4.

1, 2 only
1, 3 only
3 only
All of the above

SOLUTION:4

79. Socio-demographic changes at source that are result of rural to urban migration in India.
1. Low sex-ratio.
2. Population pyramid will shrink at its base in years to come.
3. Feminization of workforce.

Select the correct code


1.
2.
3.
4.

1, 2 only
2, 3 only
3 only
All of the above

SOLUTION: 2
Rural to urban migration mainly constitutes of male so sex ratio goes down. Even migration brings
down fertility rate so in future population pyramid will shrink.

Iasbaba.com

Page 50

IASbaba.com
80. Recently ISRO PSLV-C23 placed satellites of foreign countries, these countries include
1. France
2. Germany
3. Italy
4. Malaysia
Select the correct code
1.
2.
3.
4.

1, 2 only
1, 3 only
1, 2, 3 only
All of the above

Solution: 1
Countries include France Germany Singapore and Canada

81. Recently launched Pro-Active Governance and Timely Implementation (PRAGATI) platform
will give boost to the concept of
1, Cooperative federalism
2, Good Governance
3, Gender inclusiveness

Select the correct code


1.
2.
3.
4.

1, 2 only
2, 3 only
1, 3 only
All of the above

Solution: 1
It is a platform aimed at addressing grievances of common man. It has been rolled out to ensure
Good Governance. It will develop a special apparatus in which states and Centre both contribute. No
special mention has been made regarding gender inclusiveness but it will surely give women a voice
but not a priority basis.
Iasbaba.com

Page 51

IASbaba.com

82. Consider the statements regarding mitochondrial Donation Technique:


1. It involves addition of healthy mitochondria from donor women to an egg from other
women with defective mitochondria.
2. This technique prevents new born babies from heart failure and blindness only.
3. There are two different procedures used in mitochondrial donation technique (a) Maternal
Spindle Transfer (b) Pronuclear Transfer technique
Select the correct code
1.
2.
3.
4.

1 only
1, 2 only
1, 3 only
All of the above

Solution: 3
It prevents new born babies from brain damage, muscle wasting, heart failure and blindness due to
defective mitochondria.

http://www.independent.co.uk/life-style/health-and-families/health-news/threeparent-babieswhat-is-mitochondrial-donation-and-what-are-the-techniques-involved-10021567.html

83. Recently a cyclonic storm named Ashobaa was in news. Consider the statements
1. It was developed over east central Bay of Bengal.
2. It was named by Srilanka as there is a system of naming storms developed in Northern India
Ocean On rotational basis..
3. It caused flood in eastern Oman.
Select the correct code
1. 2 only
2. 1, 2 only
Iasbaba.com

Page 52

IASbaba.com
3. 2, 3 only
4. All of the above
Solution: 3
It was developed over east central Arabian Sea.
84. Consider the statements
1. Cryogenic engine provides more thrust with each kg of propellant it uses.
2. Cryogenic engine uses liquid oxygen and liquid hydrogen at very low temperature
3. Engine need low mass flow rate of both oxidizer and fuel to avoid explosion in combustion
chamber.
Select the correct code
1.
2.
3.
4.

2 only
1, 2 only
2, 3 only
All of the above

Solution: 2
Engine need high mass flow rate of both oxidizer and fuel to generate a sufficient thrust.
Independent regulators are there to check thrust control.
85. Consider the following statements with reference to hot and cold water
1. Cold water freezes faster than hot water
2. This phenomenon or effect is known as Mpemba Effect.
Select the correct code
1.
2.
3.
4.

Only 1
Only 2
Both
None

Solution- None
http//www.iflscience.com/chemistry/hot-water-freezes-faster-cold-and-now-we-know

86. Consider the following statements


A spaceship is out in interstellar space, far from all other objects and with all its rockets
turned off. It can be said that1. There is no net external force on it
Iasbaba.com

Page 53

IASbaba.com
2. The acceleration of the spacecraft is zero
Select the correct statements
1.
2.
3.
4.

Only 1
Only 2
Both
None

Solution- 3
Explanation- Physics, class 11th- Chapter 5. First law of Motion

87. Consider the statements regarding acceleration due to gravity on earth


1. For an object above earths surface at a height of x, the acceleration due to gravity
increases depending on the mass of the object
2. For the same object below earths surface at the same height, the acceleration due
to gravity also increases.
3. The acceleration due to gravity is maximum at the earths surface and decreasing
whether you go up or down.
Select the correct code
1.
2.
3.
4.

1 and 2
1, 2 and 3
Only 1
Only 3

Solution- 4
The remarkable thing about acceleration due to earths gravity is that its maximum on its
surface decreasing whether you go up or down.
NCERT-11th Physics-chapter 8

88. For a satellite or a circularly orbiting object in space, its energy will be reflected as

Iasbaba.com

Page 54

IASbaba.com
1. Total energy will always be positive
2. Its kinetic energy will be positive
3. Its potential will be negative
Select the correct code
1.
2.
3.
4.

1 and 2
2 and 3
1 and 3
1, 2 and 3

Solution- 2

A satellite is always at a finite distance from Earth hence its total energy cannot be zero or
positive. Potential energy will be negative and kinetic energy will be positive.

89. Consider the following statement regarding Bt Cotton

1. It is produced by Bacillus thuringiensis that produces bacteria that kill certain insects.
2. Bt toxin protein exist as active protoxins but once an insect ingest the inactive toxin,
it is converted into an active form of toxin.
Select the wrong statement/s
1.
2.
3.
4.

Only 1
Only 2
Both
None

Solution- 3

Explanation- It produces protein not bacteria. And it exist as inactive protoxins not active.

90. A person suffering from HIV will have to go for following tests?

Iasbaba.com

Page 55

IASbaba.com
1. PCR (Polymerase Chain Reaction)
2. Antibody Screening Tests
3. ELISA
Select the correct code
1.
2.
3.
4.

1 and 3
2 and 3
1, 2 and 3
Only 1

Solution- 3
All the tests are carried out for HIV

91. Neutrinos are one of the fundamental particles which make up the universe. Refer to
following statements about it
1. They are electrically neutral elementary particles
2. They are not affected by any kind of force and are neutral in nature
3. The most important source of its generation is radioactive decay
Select the correct code
1.
2.
3.
4.

Only 1
1 and 3
2 and 3
All

Solution- 1
They are not affected by electromagnetic forces but are affected by weak forces like gravity.
The most important source of its generation is sun.
Neutrinos do not carry any electric charge, which means that they are not affected by
the electromagnetic force that acts on charged particles, and are leptons, so they are not
Iasbaba.com

Page 56

IASbaba.com
affected by the strong force that acts on particles inside atomic nuclei. Neutrinos are
therefore affected only by the weak subatomic force and by gravity.

92. Prakash Path, a scheme launched recently is


1. For Light Emitting Diode (LED) bulbs distribution under Domestic Efficient Lightning
Programme (DELP)
2. System developed by the Centre of Development of Advanced Computing (C-DAC) is
a state of the art device that captures images and videos of red light violations with
the help of vehicle sensors, cameras and controller hardware and installed at busy
road intersections.
Select the correct code
1.
2.
3.
4.

Only 1
Only 2
Both
None

Solution- 1

93. A sinkhole is a depression or hole in the ground caused by some form of collapse of the
surface layer was in news recently. Consider the statements that lead to its formation

1. Diverting surface water from a large area and concentrating it in a single point
2. Artificially creating ponds of surface water and drilling new water wells
3. No recharge of the water table in the absence of good rains
Iasbaba.com

Page 57

IASbaba.com
4. They generally develop in karst regions,
Select the correct code
1.
2.
3.
4.

1,2 and 3
2,3 and 4
1,2 and 4
1,2,3 and 4

Solution- 4
Google

94. In Vitro Fertilization is the process of fertilization by manually combining an egg and
sperm in a laboratory dish, and then transferring the embryo to the uterus. It can be used
in which of the following cases
1.
2.
3.
4.

Male factor infertility including decreased sperm count or sperm motility


Women with ovulation disorders, premature ovarian failure, uterine fibroids
Individuals with a genetic disorder
Designer babies as in the case of Three Parent Babies

Select the correct code


1.
2.
3.
4.

1, 3 and 4
2, 3 and 4
1, 2, 3 and 4
1, 2 and 4

Solution- 3
http://americanpregnancy.org/infertility/in-vitro-fertilization/

95. Consider the following statements

Iasbaba.com

Page 58

IASbaba.com
1. National De-worming Day is an initiative focused on reducing the threat of protozoaworm infections.
2. It is targeted against intestinal parasitic worms among the children to achieve status
of being Worm-free
3. 10th February is celebrated as National De-worming day in India
Select the correct code
1.
2.
3.
4.

1 and 3
2 and 3
1, 2 and 3
Only 2

Solution- 2

http://www.financialexpress.com/article/healthcare/happening-now/health-ministrylaunches-national-deworming-day/40785/
96. Soil-transmitted helminth infections are among the most common infections
worldwide and affect the poorest and most deprived communities. The main species that
infect people are
1.
2.
3.

Roundworm
Whipworm
Hookworm

Select the correct code


1.
2.
3.
4.

1 and 3
2 and 3
1, 2 and 3
Only 1

Solution- 3
Was in news recently due to National De-worming day
http://www.who.int/intestinal_worms/en/

Iasbaba.com

Page 59

IASbaba.com

97. India successfully test fired its nuclear capable missile Prithvi-II. Consider the following
with reference to it1. It is surface-to-surface long range missile developed by DRDO
2. It is the second missile to be tested under Indias Integrated Guided Missile
Development Program.
3. It is developed under project Valiant of DRDO
Select the correct codes
1.
2.
3.
4.

1 and 3
Only 2
Only 3
Only 1

Solution- 4
Its the first missile under IGMDP and project name is DEVIL
98. Namami gange programme would implicitly strengthen the Indian compliance to
1. Convention of biological diversity.
2. Ramsar convention.
3. Convention on the Conservation of Migratory Species of Wild Animals (CMS)
Select the correct code:
1. 1 Only
2. 2 Only
3. All of the above
4.1and 3 Only

Solution: 3

100. Match the following


1. Champions of the Earth
Iasbaba.com

Rajiv Gandhi Environment Award


Page 60

IASbaba.com
2. Clean Technology
3. Environmental Activists

Goldman Environment Prize


United Nations Environmental Program

Select the correct code


1.
2.
3.
4.

1 and 3
2 and 3
All
None

Solution- 4
Champions of the Earth- UNEP
Clean Technology Award- Rajiv Gandhi Environment Awards
Goldman Environmental Prize- Activists

101. Consider the following species


1.
2.
3.
4.

Whales
Sharks
Polar Bears
Gazelles

Which of the following species have been recently given protection under UN
Conservation of Migratory Species of Wild Animals (CMS)?
1.
2.
3.
4.

1, 2 and 3
2, 3 and 4
1, 2, 3 and 4
1, 2 and 4

Solution- 3
http://www.smh.com.au/environment/conservation/31-species-of-migratory-animalsgiven-un-protection-20141110-11js7y.html

Iasbaba.com

Page 61

IASbaba.com
102. Consider the following
1.
2.
3.
4.

Mushroom
Cocoa
Flowers
Root and tuber crops

Which of the above is part of horticulture?


1.
2.
3.
4.

1, 3 and 4
2, 3 and 4
1, 2 and 3
All

Solution- 4
IYB-chapter 4

103. Consider the following regarding fisheries production


1. India is the largest producer of fish in the world
2. India is the second largest producer of fresh water fish in the world
Select the wrong statement/s
1.
2.
3.
4.

Only 1
Only 2
Both
None

Solution- 1
India is second largest producer of fish in the world- IYB

104. Horticulture includes


1.
2.
3.
4.

Viticulture
Floriculture
Oenology
Pomology

Iasbaba.com

Page 62

IASbaba.com
Select the correct code
1.
2.
3.
4.

1, 3 and 4
2, 3 and 4
1, 2 and 4
All

Solution- 4
Viticulture- Grapes
Floriculture- Flowers
Oenology- Making of wines
Pomology- pome fruits

105. Consider the following statements


1. The earliest treatise on dance available to us is Bharat Muni's Natyashastra, the
source book of the art of drama, dance and music.
2. The Natyashastra is also known as the fifth veda
Select the correct code
1.
2.
3.
4.

Only 1
Only 2
Both
None

Solution- 3
CCRT Wesbite- Performing arts

106. Match the following


1. Ramman
2. Mudiyettu
Iasbaba.com

Rajasthan
Ritual Theatre of Kerala
Page 63

IASbaba.com
3. Kutiyattam
4. Kalbelia folk songs and dances

Sanskrit Theatre
Religious festival of Garhwal

Chose the correct code


1.
2.
3.
4.

1, 3 and 4
2 and 3
1 and 4
All

Solution- 2
Ramman is religios festival of Garhwal and Kalbelia folk songs and dances belongs to
Rajasthan- Ministry of Culture

107. Which of the following vaccines can be given to a new born?


1.
2.
3.
4.

BCG
DPT
OPV
Hepatitis B

Select the correct code


1.
2.
3.
4.

1, 3 and 4
2, 3 and 4
1, 2 and 3
1, 2, 3 and 4

Solution- 1
1. BCG (Bacillus Calmette Guerin) 1 dose at Birth (upto 1 year if not given earlier)
2. DPT (Diphtheria, Pertussis and Tetanus Toxoid) 5 doses; Three primary doses at
6weeks,10weeks and 14 weeks and two booster doses at 16-24 months and 5 Years
of age
Iasbaba.com

Page 64

IASbaba.com
3. OPV (Oral Polio Vaccine) 5 doses; 0 dose at birth, three primary doses at 6,10 and
14 weeks and one booster dose at 16-24 months of age
4. Hepatitis B vaccine 4 doses; 0 dose within 24 hours of birth and three doses at 6, 10
and 14 weeks of age.
5. Measles 2 doses; first dose at 9-12 months and second dose at 16-24months of age
6. TT (Tetanus Toxoid) 2 doses at 10 years and 16 years of age
7. TT for pregnant woman two doses or one dose if previously vaccinated within 3
Year

108. Mission Indradhanush is a nationwide initiative with an aim to cover all those
children who are partially vaccinated or unvaccinated. It will provide protection against1.
2.
3.
4.
5.

Diphtheria,
Whooping Cough
Tetanus
Polio
Tuberculosis

Select the correct code


1.
2.
3.
4.

1, 2, 3 and 5
2, 4 and 5
1, 2 and 3
All

Solution- 4
http://pib.nic.in/newsite/efeatures.aspx?relid=105411

109. Consider the following regarding Telegraphy


1. The first telegraph came in the form of Optical telegraph.
2. In India, it became available for public use in 1854.
3. Continuous wave is used in wireless telegraphy

Iasbaba.com

Page 65

IASbaba.com
Select the correct code
1.
2.
3.
4.

1 and 3
2 and 3
1 and 2
1, 2 and 3

Solution- 4

Wireless telegraphy is still used widely today by amateur radio hobbyists where it is
commonly referred to as radio telegraphy, continuous wave, or just CW.
Telegraph services in India date back to 1850, when the first experimental telegraph line
was established between Calcutta and Diamond Harbour. The British East India Company
started using the telegraph a year later, and by 1854when the system opened to the
publictelegraph lines had been laid across the country. The telegraph continued to thrive,
in India and around the world, even after Alexander Graham Bell patented the telephone in
1876. For more than half a century, telegrams were sent over cable lines, but in 1902
(capitalizing on the work of Italian inventor Guglielmo Marconi) the Indian system went
wireless.
The first telegraphs came in the form of optical telegraph, including the use of smoke
signals, beacons or reflected light, which have existed since ancient times.

110. Project Arrow is associated with


1.
2.
3.
4.

Department of Post
Maritime exercise between France and India
DRDO project
A scientific research project under CSIR

Solution- 1
Project Arrow, an initiative to transform Department of Posts into a vibrant and responsive
organization and to make a visible and positive difference to quality and efficiency of service
to the customers, was conceived in April 2008.
Iasbaba.com

Page 66

IASbaba.com

111. Consider the following statements regarding Post Office Saving Banks in India
1. The Department of Posts operates Small Savings on behalf of Ministry of Rural Development
2. Post office saving banks do not offer ATM services
3. The Post Office Savings Bank operates Savings Account, Recurring Deposit (RD) and
Time Deposit (TD) apart from other services.
Select the correct code
1.
2.
3.
4.

Only 1
Only 2
Only 3
1, 2 and 3

Solution- 3
Comes under Ministry of Finance and offers ATM services as well
http://www.thehindu.com/news/national/tamil-nadu/indias-first-post-office-savings-bankatm-inaugurated-in-chennai/article5732624.ece

112. Consider the following


1. The territorial sea is regarded as the sovereign territory of the state, although
foreign ships (both military and civilian) are allowed innocent passage.
2. The difference between the territorial sea and the exclusive economic zone is that
the first confers full sovereignty over the waters, whereas the second is merely a
"sovereign right" which refers to the coastal state's rights below the surface of the
sea.
3. The EEZ is 188 nautical miles from Territorial waters and includes contiguous zone.

Select the correct code


1. 1 and 2
2. 2 and 3
3. Only 1
Iasbaba.com

Page 67

IASbaba.com
4. 1, 2 and 3
Solution- 4
From Base line
Territorial waters- 12 Nautical Miles
Contiguous zone- 24 Nautical Miles
EEZ- 200 Nautical Miles
From Territorial Waters- EEZ will be 188 Nautical Miles an includes Contiguous Zones
Territorial waters are also called Territorial sea.
https://en.wikipedia.org/wiki/Exclusive_economic_zone#Definition

113. Consider the following


1. Preservation and protection of Maritime Environment
2. Collection of Scientific Data
3. Protection and assistance to fishermen at sea
Which of the above includes the functions of Indian Coast Guard?
1.
2.
3.
4.

1 and 3
1 and 2
1, 2 and 3
Only 3

Solution- 3

Functions of Coast Guard as per Coast Guard Act 1978


(1) Safety and protection of artificial islands and offshore terminals installations and devices
in Maritime Zones.
Iasbaba.com

Page 68

IASbaba.com
(2) Protection and assistance to fishermen at sea while in distress.
(3) Preservation and protection of marine environment.
(4) Prevention and control of marine pollution.
(5) Assistance to customs and other authorities in anti-smuggling operations.
(6) Enforcing of enactments being in force in the maritime zones.
(7) Other matters, including measures for the safety of life and property at sea and
collection of scientific data.

114. India successfully launched its first subsonic cruise missile named Nirbhay. Consider
the following w.r.t it
1. Sub-sonic cruise missiles are able to fly at very low altitudes, close to the earth,
without being detected
2. Nirbhay blasts off like a rocket, but then unlike a missile, it turns into an aircraft.
3. Like other ballistic missiles Nirbhay also has wings and pronounced tail fins.
Select the correct codes
1.
2.
3.
4.

1 and 3
2 and 3
1 and 2
1, 2 and 3

Solution- 3
Nirbhaya has wings and pronounced tail fins unlike other ballistic missiles . So third
statement is wrong.

115. Consider the following about Guru Poornima


1. It is an Indian festival celebrated by Hindus, Jains and Buddhists
2. The first full moon after Summer Solstice is known as Guru Poornima.
Iasbaba.com

Page 69

IASbaba.com
3. Lord Shiva, the first yoga practitioner (Adi Yogi) is said to have begun imparting the
knowledge of yoga to the rest of mankind on this day and became the first guru (Adi
Guru).
Select the correct codes
1.
2.
3.
4.

1 and 2
2 and 3
1 and 3
1, 2 and 3

Solution- 4
https://en.wikipedia.org/wiki/International_Yoga_Day
https://en.wikipedia.org/wiki/Guru_Purnima

116. The terms Pushkaram and Amrapali are related with


1.
2.
3.
4.

Bharatnatyam
Kuchipuddi
Kathakali
Odissi

Solution- 2
http://www.thehindu.com/features/friday-review/dance/tales-throughballets/article7379291.ece

117. Consider the following regarding Game theory

Iasbaba.com

Page 70

IASbaba.com
1. Its based on strategic decision making
2. It finds application in psychology, economics, mathematics and biology also.
3. It finds immense potential in Artificial Intelligence
Select the correct code
1.
2.
3.
4.

1 and 3
2 and 3
1 and 2
1, 2 and 3

Solution- 4
Google it- Basic

118. Addition of which of the following has harmful effect in the process of filtration of
water?
1. Silver
2. Titanium Oxide
3. Carbon nanotube
Select the correct code
1.
2.
3.
4.

1 and 2
Only 2
1, 2 and 3
None

Solution- 4
All are used in Superior water filtration.

Silver- leaches in water and when the ions so released kill the bacteria by destroying the
integrity of the cell and by damaging the cell proteins and terminating the DNA replication.
Titanium oxide- also kills bacteria. Though its antibacterial property is best in the presence
of UV light, the present study did not use UV light.
Iasbaba.com

Page 71

IASbaba.com
Carbon Nanotubes- kill the bacteria through direct physical contactand the roughness of
the nanotubes also kills the bacteria.

119. Consider the following


1.
2.
3.
4.
5.

Isoniazid
Ethambutol
Streptomycin
Fluoroquinolones
p-Amino salicylic acid

Which of the above are Second-line anti-Tb drugs?


1.
2.
3.
4.

1, 3, 4 and 5
2 and 3
4 and 5
All

Solution- 3
Others are first-line

120. Consider the following


1.
2.
3.
4.
5.

Hay fever
Cancer
HIV
Parkinsons disease
Alzheimer

Which of the above diseases are as a result of inflammation?


1. 2, 4 and 5
2. 1,2,3 and 4
Iasbaba.com

Page 72

IASbaba.com
3. 4 and 5 only
4. 1, 2, 4 and 5
Solution- 4
HIV is not caused by inflammation. Rests of them are result of high or low inflammation of
tissues.

121. The term Biological Passport is associated with


1.
2.
3.
4.

Genetic profiling of an individual


An Athletes biological marker for Doping Test
Biological marker of a diseased person
A medical certificate for diagnosis on foreign hospitals

Solution- 2

122. Consider the following


1. Mutation
2. Gene Transfer
3. Use of antibiotics in agriculture
Which of the above will have negative impact on Anti-microbial Drug Resistance?
1.
2.
3.
4.

1 and 2
2 and 3
1 and 3
1, 2 and 3

Solution- 4
Most microbes reproduce by dividing every few hours, allowing them to evolve rapidly and
adapt quickly to new environmental conditions. During replication, mutations arise and
Iasbaba.com

Page 73

IASbaba.com
some of these mutations may help an individual microbe survive exposure to an
antimicrobial. Thus, negative impact on the anti-microbial drug resistance.
Microbes also may get genes from each other, including genes that make the microbe drug
resistant.
Scientists also believe that the practice of adding antibiotics to agricultural feed promotes
drug resistance. More than half of the antibiotics produced in the United States are used for
agricultural purposes. This makes human prone to indirect intake of such antibiotics and
helps in more active microbial activities and drugs will become more resistant lowering the
anti-microbial drug resistance.

123. Commercial Electric Motors do not use1.


2.
3.
4.

A soft iron core on which the coil is wound


A permanent magnet to rotate the armature
An electromagnet to rotate the armature
Large number of turns of the conducting wire in the current-carrying coil.

Solution- 2
NCERT-Class 10th, page number- 233

124. Consider the following statements


1. Most power stations constructed these days produce Direct Current (DC)
2. An important advantage of DC over AC is that electric power can be transmitted over
long distance without much loss of energy.
3. The use of AC over DC is the reason for power loss during transmission in Indian
power generation system.

Select the correct code


1. 1, 2 and 3
Iasbaba.com

Page 74

IASbaba.com
2. Only 1
3. 1 and 2
4. None

Solution- 4
NCERT- Class 10th- Chapter 13, page-236-37

125. The most important safety method for protecting electrical appliances from
overloading at home is1.
2.
3.
4.

Stabilizers
Electric Fuse
Earthing
Electrical Meter

Solution- 2

126. Consider the following statements


1. The conductivity of an alloy is generally higher than that of its constituent metals and
that is why they are used in most of the heating devices like Electrical Iron, Toasters
etc.
2. Diamond has more conductivity than graphite
Select the correct code
1.
2.
3.
4.

Only 1
Only 2
Both
None

Solution- 4

Iasbaba.com

Page 75

IASbaba.com
Resistivity of an alloy is higher not the conductivity. If conductivity will be higher, they will
never be used in heating devices. Graphite has more conductivity because of its shape.
Diamond has less free electronic movement than graphite so conductivity is less.

127. A student sitting on the last bench can see the letters written on the board but has
problem reading from his note book. What can be said about him?

1.
2.
3.
4.

The near point of his eyes has receded away


The farther point of his eyes has come close.
The near point of his eyes has come close
The farther point of his eyes has receded away

Solution- 1

He is suffering from Hypermetropia- far sightedness. NCERT-Class 10th- page-189-90

128. The phenomenon of rainbow formation includes


1. Dispersion
2. Total Internal Reflection
3. Refraction
Select the correct code
1.
2.
3.
4.

1 and 3
2 and 3
1 and 2
1, 2 and 3

Solution- 4
NCERT- Class 10th- page-194

Iasbaba.com

Page 76

IASbaba.com

129. Consider the following regarding sunrise and sunset


1. The time difference between actual sunrise and sunset is 4 minutes
2. This is because of the phenomenon of Total Internal Reflection
3. The apparent flattening of the Suns disc at sunrise and sunset is due to atmospheric
refraction.
Select the wrong codes
1.
2.
3.
4.

Only 1
Only 3
1 and 2
1, 2 and 3

Solution- 3
Time difference is 2 minutes and it is due to atmospheric refraction
130. At Noon, the Sun appears white because
1.
2.
3.
4.

A little of the blue and violet colors are scattered.


Blue color is scattered the most
Red colour is scattered the most
All the colours are scattered away

Solution- 1
NCERT- class 10th, page-197
131. Consider the following statements regarding GAGAN
1. Gagan is an indigenous navigational guide system developed by ISRO and Airport
Authority of India.
2. India is the third country after USA and EU to adopt this system
3. GAGAN will use a satellite based Wide Area Augmentation System (WAAS)
technology developed by the European Geostationary Navigation Overlay Service.

Select the correct code


Iasbaba.com

Page 77

IASbaba.com
1.
2.
3.
4.

1, 2 and 3
Only 2
Only 1
Only 3

Solution- 3

India is 4th country after USA, EU and Japan. GAGAN will use a satellite based Wide Area
Augmentation System (WAAS) technology developed by the Raytheon for the US Federal
Aviation Authority (FAA.).

132. Ocean Thermal Energy is due to


1.
2.
3.
4.

Energy stored by waves in the ocean


Tides arising out in the ocean
The temperature difference at different levels in the ocean
The pressure difference at different levels in the ocean

Solution- 3
The water at the surface of the ocean or sea is heated by the Sun while the water in deeper
sections is relatively cold. This difference in temperature is exploited to obtain energy in
ocean-thermal-energy conversion plants

133. Consider the following statements


1. Advanced Heavy Water Reactor is Indias third stage Nuclear programme
2. Thorium will be used as fuel in AWHR
3. Globally, Thorium is more abundant than Uranium

Select the correct option

Iasbaba.com

Page 78

IASbaba.com
1.
2.
3.
4.

1 and 2
1, 2 and 3
2 and 3
1 and 3

Solution- 2
https://en.wikipedia.org/wiki/Advanced_heavy-water_reactor

134. Exploration of Shale gas in India is fraught by several issues. Consider the statements
regarding this
1. It will severely affect ground water
2. It will give rise to release of green house gases
3. Hydraulic fracturing causes millions of very small and localized seismic events when
the fractures are actually produced in the shale
Select the correct options
1.
2.
3.
4.

1, 2 and 3
2 and 3
1 and 3
Only 3

Solution- 1
All of them are correct

Iasbaba.com

Page 79

IASbaba.com
135. Hydrogen Fuel cells are emerging as fuel of the future due its potential to address
energy crisis. Consider the statements regarding this
1. Hydrogen fuel cells converts chemical energy into electrical energy
2. They are more environment friendly as compared to other fuels like Coal and Natural
gas etc.
3. The only concern is release of CO2 and excess heat after the reaction.
Select the correct codes
1.
2.
3.
4.

Only 3
1 and 2
2 and 3
1, 2 and 3

Solution- 2
The by-product is water and heat not carbon dioxide.

136. Consider the following regarding Coal Gasification


1. Coal gasification produces syngas
2. Coal gasification can be used for making ammonia and methanol
3. Coal gasification produces less carbon dioxide than tradition coal plants
Select the correct options
1.
2.
3.
4.

1 and 2
2 and 3
1 and 3
1, 2 and 3

Iasbaba.com

Page 80

IASbaba.com
Solution- 1
The major problem in Coal Gasification is the production of more Carbon Dioxide. This was
recently in news due to Chinas aggressive use of this technology that is good for local
environment but globally the China will be adding more CO2.

136. Consider the following


1.
2.
3.
4.
5.
6.

Carbon Dioxide
Methane
Hydrogen
Carbon Monoxide
Nitrogen
Sulphur Dioxide

Which of the above are constituent of SYNGAS?


1.
2.
3.
4.

1, 2, 4 and 5
1, 2, 3, 4 and 5
3, 4, 5 and 6
2, 3, 4, 5 and 6

Solution- 2
Sulphur dioxide is not a constituent of syngas. We have asked constituents so nitrogen is
present as per most references. If question would be syn gas comprises of? Then No
Nitrogen

137. India has gifted a vintage Dakota Aircraft to


1.
2.
3.
4.

Pakistan
Bangladesh
Bhutan
Nepal

Solution- 2

Iasbaba.com

Page 81

IASbaba.com

138. Consider the following


1.
2.
3.
4.
5.

Oxidation
Hydration
Carbonation
Hydrolysis
Reduction

Which of the above are parts of chemical weathering?


1.
2.
3.
4.

1, 3 and 5
2, 4 and 5
1, 2, 3, 4
1, 3 and 4

Solution- 3
Chemical weathering- Carbonation, Oxidation and Hydrolysis
There are different types of chemical weathering, the most important are:
Solution - removal of rock in solution by acidic rainwater. In particular, limestone is
weathered by rainwater containing dissolved CO2, (this process is sometimes called
carbonation).
Hydrolysis - the breakdown of rock by acidic water to produce clay and soluble salts.
Hydration- Chemical combination of water molecules with a particular substance or mineral
leading to a change in structure.
Oxidation - the breakdown of rock by oxygen and water, often giving iron-rich rocks a rustycoloured weathered surface.

Iasbaba.com

Page 82

IASbaba.com

139. Consider the following regarding Li-ion Batteries


1. Lithium-ion batteries, unlike other rechargeable batteries, have a potentially
hazardous pressurised flammable electrolyte.
2. Li-ion batteries contain less toxic metals than other types of batteries which may
contain lead or cadmium.
3. The waste of Li-ion Batteries is generally categorized as non-hazardous waste.
Which of the above statement is/are NOT correct?
1.
2.
3.
4.

1, 2 and 3
2 and 3
1 and 2
All correct

Solution- 4

Li-ion batteries are less toxic. There were several incidences of fire in Boeing where these
batteries were used. So statement 1 is also correct. Since, they have less toxic
ingredients/metals, so their waste is considered as Non-hazardous.

140. Arrange the following Tiger Reserves from North to South direction
1.
2.
3.
4.

Kundermukh
Bor
Pilibhit
Achanakmar

Correct sequence
1.
2.
3.
4.

4-3-2-1
3-4-2-1
1-4-3-2
2-3-4-1

Iasbaba.com

Page 83

IASbaba.com

Solution- 2
1.
2.
3.
4.

Kundermukh- Karnataka
Bor- Maharashtra
Pilibhit- UP
Achanakmar- Chattisgarh

141. Consider the following statements regarding National Green Tribunal (NGT)
1. NGT can handle cases related to The Public Liability Insurance Act, 1991
2. The Tribunal shall be bound by the procedure laid down under the Code of Civil
Procedure, 1908.
3. It is a federal legislation enacted by the Parliament of India, under India's
constitutional provision of Article 21
Which of the above is NOT correct?
1.
2.
3.
4.

1 and 2
Only 1
1, 2 and 3
Only 2

Solution- 4
The Tribunal shall not be bound by the procedure laid down under the Code of Civil
Procedure, 1908, but shall be guided by principles of natural justice.
The NGT has the power to hear all civil cases relating to environmental issues and questions
that are linked to the implementation of laws listed in Schedule I of the NGT Act. These
include the following:

1.
2.
3.
4.

The Water (Prevention and Control of Pollution) Act, 1974;


The Water (Prevention and Control of Pollution) Cess Act, 1977;
The Forest (Conservation) Act, 1980;
The Air (Prevention and Control of Pollution) Act, 1981;

Iasbaba.com

Page 84

IASbaba.com
5. The Environment (Protection) Act, 1986;
6. The Public Liability Insurance Act, 1991;
7. The Biological Diversity Act, 2002.

This means that any violations pertaining only to these laws, or any order / decision
taken by the Government under these laws can be challenged before the NGT.
Importantly, the NGT has not been vested with powers to hear any matter relating to
the Wildlife (Protection) Act, 1972, the Indian Forest Act, 1927 and various laws enacted
by States relating to forests, tree preservation etc. Therefore, specific and substantial
issues related to these laws cannot be raised before the NGT. You will have to approach
the State High Court or the Supreme Court through a Writ Petition (PIL) or file an
Original Suit before an appropriate Civil Judge of the taluk where the project that you
intend to challenge is located.

142. Consider the following statements


1. The constitution of National Park in India can only be done by Central Government
2. Abolition of National Park can be done by State Legislation
Select the correct codes
1.
2.
3.
4.

Only 1
Only 2
Both
None

Solution- 2
In most countries, only national or federal government can constitute national parks, but in
India, state governments have the power to constitute national parks. In fact, national parks
can only be constituted or abolished through state legislation, whereas wild sanctuaries are
created through an order of a competent authority, who can be the chief conservator of
forests or a minister of a state. Thus, it is evident that national parks have greater degree of
permanency than a wildlife sanctuary
143. Consider the following
1.
2.
3.
4.

Markhor
Brown palm civet
Yellow Fronted Barbet
Ovaria Lurida

Iasbaba.com

Page 85

IASbaba.com

Which of the above are ENDEMIC species of INDIA?


1.
2.
3.
4.

1, 2 and 3
2, 3 and 4
1, 2, 3 and 4
2 and 4

Solution- 4
Yellow Fronted barbet is endemic to Srilanka, not India.
144. Consider the statements about Super-Heavy Elements recently in news
1. Super-heavy elements are elements beyond atomic number 104.
2. They are artificially created but do exist naturally
3. India is part of development of super heavy element 117
Select the Wrong Statement/s
1.
2.
3.
4.

1 and 2
Only 1
Only 2
Only 3

Solution- 3
They do not exist in nature.

145. Recently a Destroyer Warship was inducted by Indian Navy named INS Vishakhapatnam
Consider the statements w.r.t it

1. It is Indias first fully indigenous warship


2. Total Atmosphere Control System will enable the crew to operate the ship in regions
of nuclear fallout but provides no defence for biological or chemical fallout.
3. It is based on stealth technology and can launch supersonic missiles also
Iasbaba.com

Page 86

IASbaba.com

Select the WRONG statement/s


1.
2.
3.
4.

1 and 2
Only 3
1 and 3
2 and 3

Solution- 1
It is indigenous but partly. It will provide defence from nuclear, chemical and biological fallouts.

146. Consider the following statements based on UN report on E-waste

1. The bulk of global e-waste constitutes from mobile phones, calculators, personal
computers, printers, and small information technology equipment.
2. The lowest amount of e-waste per inhabitant was generated in South Asian
countries
Select the correct code/s
1.
2.
3.
4.

Only 1
Only 2
Both
None

Solution- 4
While only 7 per cent of e-waste last year was made up of mobile phones, calculators,
personal computers, printers, and small information technology equipment, almost 60 per
cent was a mix of large and small equipment used in homes and businesses, such as vacuum
cleaners, toasters, electric shavers, video cameras, washing machines, electric stoves,
mobile phones, calculators, personal computers, and lamps.
The lowest amount of e-waste per inhabitant was generated in Africa (1.7 kg/inhabitant).
The continent generated 1.9 Mt of e-waste in total.

Iasbaba.com

Page 87

IASbaba.com

147. Consider the following


1.
2.
3.
4.

Melting of Iron metal


Rusting of Iron
Bending of iron rod
Drawing a wire of iron metal

Which of the following are the examples of Physical Changes?


1.
2.
3.
4.

1, 2 and 3
1, 2 and 4
1, 2, 3 and 4
1, 3 and 4

Solution- 4
NCERT-class 9th
Rusting is a chemical change

148. A goalkeeper in a game of football pulls his hands backwards after holding the ball
shot at the goal. This enables the goal keeper to
1.
2.
3.
4.

exert larger force on the ball


reduce the force exerted by the ball on hands
increase the rate of change of momentum
decrease the rate of change of momentum

Solution- 2
NCERT- 9th science- chapter 9

Iasbaba.com

Page 88

IASbaba.com

149. A boy is whirling a stone tied with a string in a horizontal circular path. If the string
breaks, the stone
1.
2.
3.
4.

will continue to move in the circular path


will move along a straight line towards the centre of the circular path
will move along a straight line tangential to the circular path
will move along a straight line perpendicular to the circular path away from the boy

Solution- 3
Before the thread is released, the stone moves in a circular path with a certain speed and
changes direction at every point. The change in direction involves change in velocity or
acceleration. The force that causes this acceleration and keeps the body moving along the
circular path is acting towards the centre. This force is called the centripetal (meaning
centre-seeking) force. In the absence of this force, the stone flies off along a straight line.
This straight line will be a tangent to the circular path (NCERT- Class 9th, chapter 10)

150. Consider the following


1.
2.
3.
4.
5.

Jellyfish
Dogfish
Starfish
Silverfish
Lionfish

Which of the above are TRUE FISH?


1.
2.
3.
4.

1, 3, 4 and 5
3, 4 and 5
2, 3 and 4
2 and 5

Solution- 4 The strict biological definition of a fish, above, is sometimes called a true fish.
True fish are also referred to as finfish or fin fish to distinguish them from other aquatic
life harvested in fisheries or aquaculture
Iasbaba.com

Page 89

IASbaba.com
NCERT- Class 9th- Chapter 7

151. A change in the physical state can be brought about


1.
2.
3.
4.

only when energy is given to the system


only when energy is taken out from the system
when energy is either given to, or taken out from the system
without any energy change

Solution- 3
NCERT- 9th, Chapter 3

152. Two chemical species X and Y combine together to form a product P which contains
both X and Y
X + Y =P
X and Y cannot be broken down into simpler substances by simple chemical reactions.
Which of the following concerning the species X, Y and P are correct?
1.
2.
3.
4.

P is a compound
X and Y are compounds
X and Y are elements
P has a fixed composition

Select the correct codes


1.
2.
3.
4.

1, 2 and 3
1, 2 and 4
2, 3 and 4
1, 3 and 4

Solution- 4
Since X and Y cannot be broken down into simpler substances by simple chemical
reactions, they are not COMPOUNDS

Iasbaba.com

Page 90

IASbaba.com

NCERT- 9th, Chapter 2

153. Consider the following


1.
2.
3.
4.
5.

Evaporation of gas
Compression of the gas
Solubility of Gas
Expansion of the gas
Diffusion of gas

Which of the above phenomenon (in liquid medium) will increase with the increase in
Temperature?
1.
2.
3.
4.

1, 2, 3, 4 and 5
1, 4 and 5
1, 3, 4 and 5
2, 4 and 5

Solution- 2
NCERT- Class 9th, Chapter 1
In understanding the effects of temperature on the solubility of gases, it is first important to
remember that temperature is a measure of the average kinetic energy. As temperature
increases, kinetic energy increases. The greater kinetic energy results in greater molecular
motion of the gas particles. As a result, the gas particles dissolved in the liquid are more
likely to escape to the gas phase and the existing gas particles are less likely to be dissolved.
The converse is true as well. The trend is thus as follows: increased temperatures mean
lesser solubility and decreased temperatures mean higher solubility.
Le Chatelier's principle allows better conceptualization of these trends. First, note that the
process of dissolving gas in liquid is usually exothermic. As such, increasing temperatures
result in stress on the product side (because heat is on the product side). In turn, Le
Chatelier's principle predicts that the system shifts towards the reactant side in order to
alleviate this new stress. Consequently, the equilibrium concentration of the gas particles in
gaseous phase increases, resulting in lowered solubility.
Conversely, decreasing temperatures result in stress on the reactant side (because heat is
on the product side). In turn, Le Chtelier's principle predicts that the system shifts toward
the product side in order to compensate for this new stress. Consequently, the equilibrium
Iasbaba.com

Page 91

IASbaba.com
concentration of the gas particles in gaseous phase would decrease, resulting
in greater solubility.

154. Consider the following


1.
2.
3.
4.

Water, air, wind


Air, sugar, oil
Oxygen, water, sugar
Salt, juice, air

Above substances are arranged in the increasing order of forces of attraction between
their particles. Which one of them represents a correct arrangement?

Solution- 3
NCERT- Class 9th, chapter 1

155. Choose the correct statement of the following


1. Conversion of solid into vapours without passing through the liquid state is called
vapourisation.
2. Conversion of vapours into solid without passing through the liquid state is called
sublimation.
3. Conversion of vapours into solid without passing through the liquid state is called
freezing.
4. Conversion of solid into liquid is called sublimation

Solution- 2
NCERT-9th, Chapter 1

156. Weeds affect the crop plants by


1. killing of plants in field before they grow
2. dominating the plants to grow
3. competing for various resources of crops (plants) causing low availability of nutrients
Iasbaba.com

Page 92

IASbaba.com

Select the correct choice


1.
2.
3.
4.

1, 2 and 3
Only 2
Only 3
Only 1

Solution- 3
NCERT- Class 9th, chapter 15
157. Which of the following are Sedimentary Basins of India?
1.
2.
3.
4.

Kashmir valley
Jaisalmer
Cambay
Gondwana

Select the correct choice


1.
2.
3.
4.

1, 2 and 3
2, 3 and 4
1, 2, 3 and 4
1, 3 and 4

Solution- 3
All are sedimentary Basins
158. Which of the following departments come under Ministry of Finance?
1.
2.
3.
4.
5.

Disinvestment
Revenue
Expenditure
Consumer Affairs
Commerce

Iasbaba.com

Page 93

IASbaba.com

Select the correct code


1.
2.
3.
4.

1, 2, 3 and 5
2, 3, 4 and 5
1, 2 and 3
3, 4 and 5

Solution- 3

159. Consider the following statements


1. Phillips curve is direct relation of unemployment and inflation.
2. It explains deflation and hyperinflation much better than stagflation
Which of the above is/are NOT correct?
1.
2.
3.
4.

Only 1
Only 2
Both
None

Solution- 3
Inverse relation between unemployment and inflation- Phillips curve
Stagflation is the combination of high unemployment and economic stagnation with
inflation.

160. Consider the following statements


1. The highest population of Scheduled Tribes in India lives in Andhra Pradesh
2. There are more Scheduled Tribes in India than Scheduled Castes

Iasbaba.com

Page 94

IASbaba.com

Select the correct statement/s


1.
2.
3.
4.

Only 1
Only 2
Both
None

Solution- 4
Highest population of STs- MP
More SCs than STs

161. Heinz Award is given in the field of


1.
2.
3.
4.
5.

Art
Environment
Public Policy
Mathematics
Technology

Select the correct code


1.
2.
3.
4.

1, 2, 3, 4 and 5
2, 3, 4 and 5
1, 2, 3 and 5
1, 2, 4 and 5

Solution- 3
It is awarded in - Arts and Humanities; Environment; Human Condition; Public Policy; and
Technology, the Economy and Employment.
162. Consider the statements
1. West Bank is more close to Mediterranean Sea while Gaza Strip is far
2. Gaza strip is more close to Mediterranean Sea than Dead sea
3. West Bank is close to Dead Sea than Mediterranean Sea
Iasbaba.com

Page 95

IASbaba.com
4. Gaza Strip is close to Dead Sea than Mediterranean Sea

Identify the correct statement/s


1.
2.
3.
4.

1 and 4
2 and 3
1 and 3
2, 3 and 4

Solution- 2
GAZA Strip is adjacent to Mediterranean Sea and West Bank is close to Dead Sea

163. Consider the following regarding Competition Appellate Tribunal of India


1. It is a non-statutory body under Competition Commission of India
2. They have the power similar to Civil Courts
3. The Appellate Tribunal shall not be bound by the procedure laid down in the Code of Civil
Procedure, 1908 but shall be guided by the principles of natural justice
Which of the above is NOT correct about CAT?
1.
2.
3.
4.

Only 3
1 and 2
Only 1
None

Solution- 3
The Competition Appellate Tribunal is a statutory organization established under the provisions of
the Competition Act, 2002 to hear and dispose of appeals against any direction issued or decision
made or order passed by the Competition Commission of India.
The Appellate Tribunal shall not be bound by the procedure laid down in the Code of Civil Procedure,
1908 (5 of 1908), but shall be guided by the principles of natural justice and, subject to the other
provisions of this Act and of any rules made by the Central Government. The Appellate Tribunal shall
have, for the purposes of discharging its functions under the Act, the same powers as are vested in a
civil court under the Code of Civil Procedure, 1908.

Iasbaba.com

Page 96

IASbaba.com

164. Joint Comprehensive Plan of Action, a Nuclear deal was


1.
2.
3.
4.

Nuke deal between US, UN and Iran


Civilian Nuke deal Agreement between UN, P+5 and Iran
Signed in Vienna, between Iran and US, Russia, China, Germany, France Plus EU
Signed in Vienna, between Iran and P+5 plus Germany and the EU.

Solution- 4

165. IRENA Headquarter located in


1.
2.
3.
4.

Tel Aviv City, Israel


Madaba City, Jordan
Masfut City, UAE
Masdar City, UAE

Solution- 4

166. Match the following


City
1.
2.
3.
4.

River

Hanoi
Bangkok
Phnom-Penh
Yangon

(a) Menam
(b) Irrawaddy
(c) Mekong
(d) Red River

Codes

1.
2.
3.
4.

a
b
d
d

b
c
a
a

c
a
c
b

d
d
b
c

Iasbaba.com

Page 97

IASbaba.com
Solution- 3

167. in International law, what does Mare clausum refers to?

1. An area in international waters where ocean mining is prohibited


2. A sea, ocean or other navigable body of water under the jurisdiction of state that is open to
navigation to ships of all nations.
3. A sea, ocean or other navigable body of water under the jurisdiction of a state that is closed
or not accessible to other states.
4. Both (1) and (2)

Solution- 3
168. Consider the following with respect to Deep Sea Adaptation
1. The species have high agility than surface water species
2. The species have reflective scales
3. The species tend to grow much faster than surface sea species.
Select the correct statement/s
1.
2.
3.
4.

1 and 2
Only 2
Only 3
1, 2 and 3

Solution- 2, Statements 1 and 3 are wrong. They have low agility. They grow much slower than
surface sea species.

169. Which of the following statement is not true for organic manure?
1.
2.
3.
4.

It enhances water holding capacity of soil.


It has a balance of all plant nutrients.
It provides humus to soil.
It improves texture of soil

Solution- 2
Self explanatory. NCERT- class 8th, chapter -1

Iasbaba.com

Page 98

IASbaba.com

170. Read the statements given below.


1.
2.
3.
4.

Seeds require moisture for germination.


Plants can absorb nutrients mostly in dissolved form.
Irrigation protects crops from both frost and hot air currents.
Irrigation improves soil texture.

Choose the combination of statements which indicate the need to irrigate crops.
1.
2.
3.
4.

1, 2 and 3
2, 3 and 4
1, 2 and 4
1, 2, 3 and 4

Solution- 1
4 is wrong. Irrigation has no relation with texture of the soil.
NCERT- class 8th, chapter -1

171. Given below are statements about the effects of weeds on crop plants.
1.
2.
3.
4.

They interfere in harvesting.


They help crop plants to grow healthily.
They compete with crop plants for water, nutrients, space and light.
They affect plant growth.

Choose the correct combination of statements.


1.
2.
3.
4.

1, 3 and 4
3 only
3 and 4
1, 2, 3 and 4

Solution- 1
How will weed help crop plants grow healthily? Eliminate it and answer is 1
Iasbaba.com

Page 99

IASbaba.com
NCERT- class 8th, chapter -1

172. Consider the following diseases


1.
2.
3.
4.

Malaria
TB
Cold Cough
Chicken Pox

Which of them are NOT caused by Virus?


1.
2.
3.
4.

2, 3 and 4
1 and 2
1, 3 and 4
3 and 4

Solution- 2
Cold Cough and Chicken pox are caused by Virus. Malaria by Protozoans and TB is caused by
Bacteria
NCERT- class 8th, chapter -2

173. Micro-organisms around us are useful as well as harmful for us. In this light consider
the following statements
1. They can increase the fertility of the soil
2. They helps in nitrogen fixation
3. They spoil our food hence not used as food additives
Select the Wrong statement/s
1.
2.
3.
4.

1 and 3
Only 3
Only 2
1, 2 and 3

Solution- 2

Iasbaba.com

Page 100

IASbaba.com
Some bacteria and blue green algae are able to fix nitrogen from the atmosphere to enrich
soil with nitrogen and increase its fertility. These microbes are commonly called biological
nitrogen fixers.

Many microorganisms derived substances are used as food additives and for fermentation
purposes like Bakers Yeast.
NCERT- class 8th, chapter -2

174. Given below are events that lead to pregnancy and development of embryo.
1.
2.
3.
4.

Fertilization of egg
Maturation of egg
Release of egg
Embedding of embryo in thickened uterine wall.

Which of the following options gives the correct order of sequence in which they occur?

1.
2.
3.
4.

1, 2, 3, 4
2, 1, 3, 4
1, 4, 2, 3
2, 3, 1, 4

Solution- 4
NCERT-8th, chapter 10

175. Match items in List A with the items of list B.


List A
1. Nylon
2. PET
3. Rayon
Iasbaba.com

List B
(i) non-stick coating
(ii) electric switches
(iii) parachutes
Page 101

IASbaba.com
4. Thermosetting plastics
5. Teflon

(iv) polyester
(v) artificial silk

Code
1.
2.
3.
4.

1-iii, 2-iv, 3-v, 4-ii, 5-i


1-i, 2-ii, 3-iii, 4-iv, 5-v
1-ii, 2-i, 3-iv, 4-v, 5-iii
1-iv, 2-iii, 3-v, 4-ii, 5-i

Solution- 1
NCERT-Class 8th, chapter 3

175. Consider the following


1.
2.
3.
4.
5.

paraffin wax
petrol
lubricating oil
coke
Bitumen

Which of the above is not a constituent of petroleum?


1.
2.
3.
4.

3, 4 and 5
4 and 5
Only 4
1 and 4

Solution- 3
NCERT-Class 8th, chapter 5
176. Choose the correct statement about inflammable substances from the following.
They have:
Iasbaba.com

Page 102

IASbaba.com
1.
2.
3.
4.

Low ignition temperature and cannot catch fire easily.


High ignition temperature and can catch fire easily.
Low ignition temperature and can catch fire easily.
High ignition temperature and cannot catch fire easily

Solution- 3
NCERT-Class 8th, chapter 6

177. In the list of animals given below, Hen is the odd one out.
Human Being, Cow, Dog, Hen
The reason for this is
1.
2.
3.
4.

It undergoes internal fertilisation.


It is oviparous.
It is viviparous.
It undergoes external fertilisation.

Code
1.
2.
3.
4.

1, 2 and 4
Only 3
Only 2
1 and 2

Solution- 3
Oviporous- Lay Eggs. Hens also undergo internal fertilization as others but question asked
for odd reason.
NCERT-Class 8th, chapter 9

178. World is all set to include New Sustainable Development Goals (SDGs) by the end of
2015. Consider the following statements regarding this
1. It has the element of United Nations Convention to Combat Desertification (UNCCD)
2. It is based on Agenda-21

Iasbaba.com

Page 103

IASbaba.com
3. Indias Total Sanitation Program and Swatch Bharat do not come under new set of
targets of SDGs
4. There is no mention of tackling tobacco control and illegal drug trafficking

Which of the above statement/s is/are TRUE?


1.
2.
3.
4.

1 and 2
1, 2 and 3
2 and 4
1, 2, 3 and 4

Solution- 1
3 and 4 are wrong. Read all the 17 Goals properly.

179. Consider the following


1.
2.
3.
4.
5.
6.
7.

Tropospheric Ozone
Sulphur dioxide
CO
CO2
Water vapour
Methane
Nitrous oxide

Which of the above are Short Lived Green House Gases?


1.
2.
3.
4.

1, 2, 3, 6 and 7
2, 3, 5, 6 and 7
1, 2, 3 and 5
2, 3, 5 and 7

Solution- 3

Iasbaba.com

Page 104

IASbaba.com
Short-lived gases (e.g., sulphur dioxide and carbon monoxide) are chemically reactive and
generally removed by natural oxidation processes in the atmosphere, by removal at the
surface or by washout in precipitation; their concentrations are hence highly variable.
Ozone is a significant greenhouse gas that is formed and destroyed by chemical reactions
involving other species in the atmosphere.
The amount of water vapor in the atmosphere exists in direct relation to the temperature. If
you increase the temperature, more water evaporates and becomes vapor, and vice versa.
So when something else causes a temperature increase (such as extra CO2 from fossil fuels),

more water evaporates. Then, since water vapor is a greenhouse gas, this additional water
vapor causes the temperature to go up even furthera positive feedback. The other factor
to consider is that water is evaporated from the land and sea and falls as rain or snow all the
time. Thus the amount held in the atmosphere as water vapour varies greatly in just hours
and days as result of the prevailing weather in any location. So even though water vapour is
the greatest greenhouse gas, it is relatively short-lived.
https://www.ipcc.ch/publications_and_data/ar4/wg1/en/tssts-2-1.html

180. India is moving from non-renewable source of energy to renewable energy as


evident from aggressive policies for the same. In this light consider the following
statements
1. The contribution of Solar Energy is dominant in overall renewable energy sector of
India.
2. Wind Energy dominates the sector followed by solar, biomass and hydro-power
Select the correct code/s
1.
2.
3.
4.

Only 1
Only 2
Both
None

Solution- 4
Wind Energy dominates with 66% of installed capacity followed by Biomass, Hydro and solar

Iasbaba.com

Page 105

IASbaba.com
181. India has established National Adaptation Fund following the international norm of
Adaptation Fund under Climate Change Agreement. Consider the following statements
regarding this
1. The Adaptation Fund was established in 2007 at the 7th COP of UNFCCC.
2. The Adaptation Fund is financed with a share of proceeds from the Joint
Implementation Mechanism (JIM) project activities and other sources of funding.
3. The Industrial Development Bank of India (IDBI) is Indias National Implementing
Entity (NIE) for the Adaptation Fund created under the UNFCCC.

Select the WRONG statement/s


1.
2.
3.
4.

1 and 2
1 and 3
2 and 3
1, 2 and 3

Solution- 4
The Adaptation Fund was established in 2001 but launched in 2007. It proceeds from Clean
Development Mechanism (CDM) and NABARD is Indias National Implementing Entity not
IDBI

182. Consider the following entities


1.
2.
3.
4.
5.
6.

Edible Oil and Seeds


Cotton
Petroleum products
Drugs
Fertilizers
Sugars

Which of the above commodities are covered in Essential Commodity Act, 1955 of GoI?
1.
2.
3.
4.

2, 3, 4, 5 and 6
2, 3, 4 and 6
1, 3, 5 and 6
1,2, 3, 4, 5 and 6

Iasbaba.com

Page 106

IASbaba.com
Solution- 4

183. Consider the following statements


1. A Swamp is a Wetland that is forested
2. Myristica swamps are tropical fresh water swamp forests found in Western Ghats
3. Swamps serve vital roles in flood protection and nutrient removal
Select the correct code
1.
2.
3.
4.

1 and 2
2 and 3
1 and 3
1, 2 and 3

Solution- 3
http://water.epa.gov/type/wetlands/swamp.cfm
http://www.thehindu.com/sci-tech/energy-and-environment/myristica-swamps-avanishing-ecosystem-in-western-ghats/article3512630.ece

184. Consider the following regarding PSLV and GSLV


1. PSLV are used to place remote sensing satellites to Sun-synchronous polar orbits
only while GSLV can place satellites into Geosynchronous Transfer Orbit (GTO)
2. GSLV is more powerful and reliable launch vehicle than PSLV
3. GSLV has four stage cryogenic system while PSLV has three stage solid and liquid
propulsion system
Identify the Correct Statement/s
1. 1, 2 and 3
Iasbaba.com

Page 107

IASbaba.com
2. 2 and 3
3. 1 and 2
4. None

Solution- 4
PSLV can also launch satellites into GTO. PSLV is more reliable than GSLV. GSLV has three
stages while PSLV has four stages.

185. WiMAX and Wi-Fi are wireless technologies having similar features. Consider the
following statements regarding them
1. WiMAX provides wireless broadband connectivity for short ranges while Wi-Fi
provides for long ranges
2. Wi-Fi is more controlled and requires a licensed spectrum for official and Home
networks, provided by service operator whereas WiMAX can be used in small offices
and Home without licensed spectrum.
3. WiMAX provides more stronger frequency than Wi-Fi
Identify the WRONG statements
1.
2.
3.
4.

1, 2 and 3
2 and 3
1 and 3
None

Solution- 1

1. WiMAX stands for Worldwide Interoperability for Microwave Access; Wi-Fi stands
for Wireless Fidelity.

2. WiMAX provides wireless broadband connectivity for long ranges; Wi-Fi provides shortrange, wireless broadband connectivity mostly within an office or home.

3. WiMAX is more controlled and requires a licensed spectrum; the service is deployed by
Iasbaba.com

Page 108

IASbaba.com
the service providers. Wi-Fi can work in a less controlled environment; it works in an
unlicensed environment and is less controlled. Moreover, the end users have to buy the
devices.
4. WiMAX uses MAC protocol which is connection oriented; Wi-Fi uses connection-based or
connectionless protocol called CSMA/CA.

186. Consider the following statements about Green Diesel and Bio Diesel
1. They are second generation biofuels
2. Green diesel has different chemical properties as compared to petroleum-based
diesel
3. Green Diesel is produced by Trans-esterification while Biofuels are produced by
hyrdrogenating triglycerides
Identify the correct statement/s
1.
2.
3.
4.

1, 2 and 3
2 and 3
1 and 3
None

Solution- 4
They are first generation biofuels. They have similar chemical properties. In third statement,
Trans-esterification- Biofuels and Hydrogenation- Green Diesel

187. Recently Japanese Encephalitis (JE) has claimed many lives in Assam, India. Consider
the following statements regarding this
1. It is a viral disease caused by a flavivirus.
2. JE is related to Yellow fever, West Nile viruses and Dengue
3. Birds and Pigs are host
Iasbaba.com

Page 109

IASbaba.com
Identify the WRONG statement/s
1.
2.
3.
4.

1, 2 and 3
2 and 3
1 and 2
None

Solution- 4
All are correct.
http://www.who.int/mediacentre/factsheets/fs386/en/

188. National Youth Policy-2014 intends to empower youth. What are the policy
interventions to be followed in this regard?
1.
2.
3.
4.
5.

Promoting social values among youth


Engaging them in sports activities
Involving them in politics
Entrepreneurship skills
Giving equitable Health facilities

Select the correct codes


1.
2.
3.
4.

1, 2, 4 and 5
2, 3, 4 and 5
1, 2 and 5
1, 2, 3, 4 and 5

Solution- 4

The Policy seeks to recommend specific future policy interventions required in each of the
11 priority areas.

Iasbaba.com

Page 110

IASbaba.com
Create a productive work force
1. Education 2. Entrepreneurship 3. Employment and Skill Development
Develop a strong and healthy generation
4. Health and Healthy Lifestyle 5. Sports
Instill social values and promote community service
6. Promotion of Social Values 7. Community Engagement
Facilitate participation and civic engagement

8. Participation in Politics and Governance 9. Youth Engagement


Support youth at risk and create equitable opportunity for all
10. Inclusion 11. Social Justice

189. Consider the following statements regarding Prevention of Money Laundering Act
2002
1. To Prevent & control money laundering to confiscate & seize property.
2. PMLA was, enacted in 2002, but was amended first in 2005, then in 2009, then in
2012.
3. It extends to whole of India including J&K
4. Acts covered under PMLA includes offences related to Custom Act, Prevention of
Corruption Act, Environment Protection Act and Wildlife Protection Act.
Choose the correct options from above
1) 1, 2, 3 and 4
2) 2, 3 and 4
3) 1, 2 and 3
4) Only 3
Solution- 1
Iasbaba.com

Page 111

IASbaba.com
The offences listed in the Schedule to the Prevention of Money Laundering Act, 2002 are
scheduled offences in terms of Section 2(1)(y) of the Act. The scheduled offences are
divided into two parts - Part A & Part C. In part A, offences to the Schedule have been listed
in it comprises of offences under Indian Penal Code, offences under Narcotic Drugs and
Psychotropic Substances, offences under Explosive Substances Act, offences under Unlawful
Activities (Prevention) Act, offences under Arms Act, offences under Wild Life (Protection)
Act, offences under the Immoral Traffic (Prevention) Act, offences under the Prevention of
Corruption Act, offences under the Explosives Act, offences under Antiquities & Arts
Treasures Act etc. Part C deals with trans-border crimes, and is a vital step in tackling
Money Laundering across International Boundaries.

Acts covered under Schedule


(a)Indian Penal Code, 1860; (b) NDPS Act, 1985; (c) Unlawful Activities (Prevention) Act,
1967; (d) Prevention of Corruption Act, 1988; (e) Customs Act, 1962; (f) SEBI Act, 1992; (g)
Copyright Act, 1957; (h) Trade Marks Act, 1999; (i) Information Technology Act, 2000; (j)
Explosive Substances Act, 1908; (k) Wild Life (Protection) Act, 1972; (l) Passport Act, 1967;
(m) Environment Protection Act, 1986; (n) Arms Act, 1959.
The act extends to whole of India including J&K
190. Consider the following Statements regarding International Criminal Court and
International Court of Justice
1. ICC is an older organization than ICJ
2. Both works on international jurisdiction with UN approval
3. Both are based in Hague, Netherland
Select the correct code
1.
2.
3.
4.

1 and 3
Only 2
1, 2 and 3
Only 3

Solution- 4

Iasbaba.com

Page 112

IASbaba.com
ICJ is older than ICC. ICC is intergovernmental organization and tribunal that sits in Haque in
Netherland. And The ICJ is justice branch of UN, located in Haque, Netherland

191. Consider the following regarding Curative Petition


1. There is a time limit for filling of curative petition.
2. A curative petition is filed generally immediately after the impugned judgment is
passed, with the intention of expressing dissatisfaction and seeking an instant relook
at the same facts of the case
3. The court could impose "exemplary cost" to the petitioner if his plea lacks merits
Select the correct code/s
1.
2.
3.
4.

1, 2 and 3
1 and 2
2 and 3
Only 3

Solution- 4
In a curative petition, the petitioner is required to aver specifically that the grounds
mentioned therein had been taken in the review petition filed earlier and that it was
dismissed by circulation. This has to be certified by a senior advocate. The curative petition
is then circulated to the three senior most judges and the judges who delivered the
impugned judgement, if available. While no time limit as such is given for filing of curative
petitions, the following stringent limitations exist regarding the admissibility of such a
petition:
Iasbaba.com

Page 113

IASbaba.com
(1) There must be a clear violation of principles of natural justice in that a party was not a
party to the list but the judgment adversely affected his interests or, if he was a party to the
lis, he was not served with notice of the proceedings and the matter proceeded as if he had
notice or
(2) Where in the proceedings a Judge failed to disclose his connection with the subjectmatter or the parties giving scope for an apprehension of bias and the judgment adversely
affects the petitioner.
However, curative petitions are treated with great caution by the Supreme Court, with the
recent petition in the Bhopal case being dismissed despite the bench expressing its
sympathies with the victims, citing the misinterpretation of its former judgment by the State
as the reason for non-prosecution and no express decision to that effect as such.

All in all, the highest court of the land expressing its fallibility and then taking that forward
to correct its errors by virtue of innovative interpretative means is indeed something
commendable in a system of constitutional governance that advocates checks and balances
as a tool of accountability.
Second statement is of Review Petition
To entertain the curative petitions, the court has laid down certain specific conditions. Its
laid down in order The requirements which are needed in order to accept the curative
petitions are:
1. The petitioner will have to establish that there was a genuine violation of principles
of natural justice and fear of the bias of the judge and judgment that adversely
affected him.
2. The petition shall state specifically that the grounds mentioned had been taken in
the review petition and that it was dismissed by circulation.
3. The curative petition must accompany certification by a senior lawyer relating to the
fulfillment of the above requirements.
4. The petition is to be sent to the three senior most judges and judges of the bench
who passed the judgment affecting the petition, if available.
5. If the majority of the judges on the above bench agree that the matter needs
hearing, then it would be sent to the same bench (as far as possible).
6. The court could impose exemplary costs to the petitioner if his plea lacks merit.
Source- WIKI

Iasbaba.com

Page 114

IASbaba.com
192. Consider the following regarding melting of ice and its effects
1. It will lower down Earths temperature
2. It will affect Gravity of Earth
3. Give way to release of more green house gases
Select the correct code/s
1.
2.
3.
4.

1 and 2
2 and 3
1 and 3
1, 2 and 3

Solution- 2
Melting of ice will actually increase Earths temperature
http://www.livescience.com/48099-antarctica-melting-earth-gravity-changes.html

193. Toxins are poisonous substances and are extremely harmful. Consider the following
statements regarding this
1. They are biological agents
2. They can be created synthetically
3. Toxins can be produced from bacteria, fungi, algae and plants
Select the WRONG statement/s
1.
2.
3.
4.

1, 2 and 3
1 and 2
2 and 3
1 and 3

Solution- 2
They are not biological agents. According to an International Committee of the Red
Cross review of the Biological Weapons Convention, "Toxins are poisonous products of
organisms; unlike biological agents, they are inanimate and not capable of reproducing
Iasbaba.com

Page 115

IASbaba.com
themselves", and "Since the signing of the Convention, there have been no disputes among
the parties regarding the definition of biological agents or toxins". Toxin is
a poisonous substance produced within living cells or organisms, synthetic toxicants created
by artificial processes are thus excluded.

194. Consider the following regarding Reflecting surface of a spherical mirror


1. Centre of curvature is part of the mirror
2. The centre of curvature of a concave mirror lies behind the mirror
3. The centre of curvature of a convex mirror lies in front of it.

Select the WRONG statement/s


1.
2.
3.
4.

1, 2 and 3
2 and 3
1 and 2
None

Solution- 1
NCERT- CLASS 10th, chapter 10, page 162

195. Consider the following


1.
2.
3.
4.

Wavelength
Frequency
Size of the particle
Mass of the particle

Colour of the scattered light depends on


1.
2.
3.
4.

1 and 2
1, 2, 3 and 4
Only 3
1, 2 and 4

Iasbaba.com

Page 116

IASbaba.com
Solution- 3
Tyndal effect- colour of the scattered light depends on the size of the scattering particle.
Light scattering depends on wavelengths and frequency. NCERT- 10th chapter- 11

196. The flow of charge in a conducting metallic wire depends on


1.
2.
3.
4.

Chemical reaction
Gravity
Potential Difference
Current

Select the correct code/s


1.
2.
3.
4.

1, 3 and 4
2, 3 and 4
1, 2 and 3
1, 2, 3 and 4

Solution- 1
Gravity has no role to play. The electron moves only if there is a difference of electric
pressure called potential difference. The chemical action within the cell generates the
Potential difference across the terminal even when no current is drawn from it. Flow of
charge or net charge is product of current and time.
NCERT- class 10th chapter 12

197. Superconductors have been an exciting field of research for scientists due to its
properties that can transform the outlook of application based technologies in years to
come. Consider the following regarding this
1. In all Superconductors, magnetism and superconductivity are mutually exclusive.
2. The major problem in application of superconducting materials in daily life is the
high temperature process of heating that restricts its usage.
Iasbaba.com

Page 117

IASbaba.com
3. MAGLEV technology and Smart Grid finds application of superconductivity
Select the correct code/s
1.
2.
3.
4.

1, 2 and 3
Only 1
1 and 2
Only 3

Solution- 4
Generally in superconductors, magnetism and superconductivity are mutually exclusive but
recent developments have proved otherwise. Read this article
http://www.frontline.in/science-and-technology/unexpectedsuperconductor/article7003261.ece

Second statement is wrong since its the low temperature that is problematic. Third
statement is right.

198. With reference to Cloud computing and Grid computing consider the following
statements
1. Grid computing puts resources into one place while Cloud computing distributes it to
many locations.
2. Cloud computing is typically for many small tasks while grid computing is suited for a
few large tasks.
3. Large cloud computing instances utilize grid computing internally.
Select the correct code/s
1.
2.
3.
4.

1, 2 and 3
2 and 3
1 and 3
Only 2

Solution- 2

Iasbaba.com

Page 118

IASbaba.com
First statement is reversed. Cloud computing put resources into one place while Grid
computing distributes it to many locations.

199. Internet Service Provider and Internet Protocol are commonly used when we think
about internet. Consider the following regarding them
1. One cannot have IP without ISP
2. ISP is the provider of the internet connection while IP is a unique network identifier
3. The ISP is always the same while some subscribers can have constantly changing IPs
IPs are limited while ISPs are not
4. IPs can exist without the internet while ISPs cannot
Select the correct code/s
1.
2.
3.
4.

1, 2, 3 and 4
2, 3 and 4
1, 2 and 4
3 and 4

Solution- 2
Even without ISP, IP exists.
Rest statements are right

200. Fragile Five includes


1.
2.
3.
4.
5.
6.
7.

Nigeria
Brazil
India
Turkey
Greece
Indonesia
South Africa

Select the correct option


1. 1, 2, 3, 4 and 7
2. 2, 3, 4, 6 and 7
3. 1, 3, 4, 5, 6 and 7
Iasbaba.com

Page 119

IASbaba.com
4. 2, 4, 5, 6 and 7

Solution- 2
Fragile Five- Brazil, India, South Africa, Turkey and Indonesia

201. Macro-Economic Vulnerability Index adds up


1.
2.
3.
4.
5.

Balance of Payment
Fiscal Deficit
Rate of Inflation
Current Account Deficit
Depreciation

Select the correct code/s


1.
2.
3.
4.

1, 2, 3, 4
2, 3 and 4
1, 2, 3, 4 and 5
2, 3, 4 and 5

Solution- 2
Macroeconomic Vulnerability Index, as mentioned in the Mid-Year Economic Analysis 20142015, released by Ministry of Finance recently, adds together the rate of inflation, current
account deficit and fiscal deficit of a country. The Index value can be compared across
countries for different time periods to gauge their relative vulnerability. The ministry has
done a comparison of the fragile five countriesBrazil, India, Indonesia, South Africa and
Turkey

202. Correctly match the following committees recently in news

Iasbaba.com

Page 120

IASbaba.com

1.
2.
3.
4.

Name

Area

Shanta Kumar Committee


Meena Kumari Committee
Ramesh Chand Committe
Param Rajput

FCI
Minimum Support Price
Deep Sea Fishing
Status of Women

Correct option
1.
2.
3.
4.

1, 2 and 3
2 and 3
1 and 4
1, 3 and 4

Solution- 3
Meena Kumari Committe- Deep Sea Fishing
Ramesh Chand- MSP

202. Recently World Hepatitis Day was celebrated around the globe. Consider the
following statements regarding Hepatitis
1. It is a form of inflammatory disease
2. There are 5 form of Hepatitis Virus i.e A, B, C, D and E
3. All viruses are transmitted through consumption of contaminated water or food
Select the correct code/s
1.
2.
3.
4.

1, 2 and 3
2 and 3
1 and 3
1 and 2

Solution- 4

Only Hepatitis A and E are transmitted through consumption of contaminated water or


food.
http://www.who.int/features/qa/76/en/
Iasbaba.com

Page 121

IASbaba.com
28th July- WORLD HEPATITIS DAY
203. Arrange the following in correct chronology of launch
1.
2.
3.
4.

National Urban Livelihood Mission


Indira Awas Yojna
Jawahar Lal Nehru National Solar Mission
Swarna Jayanti Shahri Rozgar Yojna

Correct code
1.
2.
3.
4.

4-2-1-3
2-3-1-4
2-4-3-1
4-1-2-3

Solution- 3
National Urban Livelihood Mission- 2013
Indira Awas Yojna 1985

Jawahar Lal Nehru National Solar Mission- 2005


Swarna Jayanti Shahri Rozgar Yojna- 1997

204. Consider the following statements


1. National Rural Livelihood Mission has been restructured into Swamjayanti Gram
SwarozgarYojana (SGSY)
2. Recently SGSY is subsequently renamed as Aajeevika.
3. The restructuring of schemes are consequence of inefficient social mobilization of
poor, improper SHGs formation and low credit mobilization.
Select the Wrong code
1.
2.
3.
4.

1, 2 and 3
1 and 2
Only 3
1 and 3

Iasbaba.com

Page 122

IASbaba.com
Solution- 2

First two statements are interchanged. Read carefully. NRLM was as a result of restructuring
of SGSY, not otherwise. And Aajeevika is new name of NRLM. Third statement is correct

205. National Social Assistance Program of GoI is in consonance to which article of Indian
Constitution?
1.
2.
3.
4.

Article 40
Article 16
Article 335
Article 41

Solution- 4

Article 41- Right to work, to education and to public assistance in certain cases The State
shall, within the limits of its economic capacity and development, make effective provision
for securing the right to work, to education and to public assistance in cases of
unemployment, old age, sickness and disablement, and in other cases of undeserved want.
Article 41 of the Constitution of India directs the States to provide public assistance to its
citizens in cases of unemployment, old age, sickness and disablement and also in other
cases within the limit of its economic capacity and development. It is in accordance with
these noble principles that the Government of India on 15 th August, 1995 included the
National Social assistance Programme (NSAP) in the Central Budget for 1995-96.

206. Consider the following statements


1.
2.
3.
4.

Lead is a toxic non-metal


India export majority of its Lead Production
Lead is used in Painting, Manufacturing of glasses and as anti-friction agent
Rajasthan is the leading producer of Lead in India

Select the correct code/s

Iasbaba.com

Page 123

IASbaba.com
1.
2.
3.
4.

1, 2, 3 and 4
2 and 3
3 and 4
1, 2 and 3

Solution- 3
Lead is a metal. Lead production is deficient hence import. Last two sentences are correct.

207. Natural Gas and LPG are one of the cleanest sources of available Energy. Consider the
following statements regarding this
1. Natural gas has higher calorific value than LPG
2. LPG is more combustible than Natural Gas
3. Natural Gas is heavier than LPG

Select the correct code/s


1.
2.
3.
4.

1, 2 and 3
Only 3
1 and 2
None

Solution- 4
Invert the statements
There are two main differences in the way that LPG (Propane) and natural gas (Methane)
are burnt. The first difference is in the energy content.
LPG has a higher calorific value, or energy content, so less gas is required to produce the
same amount of heat. LPG Needs More Oxygen. The second difference is in the oxygen to
gas ratio required for proper combustion. LPG requires oxygen to gas ratio of approximately
25 to 1. Natural gas requires a ratio of around 10 to 1.

208. Consider the difference between Fission and Fusion reactions


Iasbaba.com

Page 124

IASbaba.com
1. Both gives chain reactions
2. Fission requires more energy than Fusion to start the reaction
3. Fusion gives more stable reaction than Fission
Select the correct code/s
1.
2.
3.
4.

1, 2 and 3
2 and 3
1 and 2
None

Solution- 4
Fission gives Chain reactions not Fusion. Fusion requires tremendous energy to combine the
H-nuclei. Controlling the Fusion (High Energy production) is a major problem in Fusion
Research. So, un-stable due to high energy generation.

209. Consider the following statements regarding Seismic gaps


1. A seismic gap is a section of a fault that has produced earthquakes in the past but is
now quiet.
2. An earthquake of high magnitude occurs around the region of seismic gap
3. They are sections of plate boundaries that have ruptured in recent past
Select the Wrong statement/s
1.
2.
3.
4.

1 and 2
2 and 3
Only 3
None

Solution- 2

210. Which of the following statement(s) with regard to the Armed Forces (Special
Powers) Act, 1958 is/are correct?
Iasbaba.com

Page 125

IASbaba.com

1. The Act is applicable to the States of Manipur, Tripura and Nagaland and J&K
2. Any person arrested and taken into custody under this Act shall be made over to the
officer in charge of the nearest police station with the least possible delay, together
with a report of the circumstances occasioning the arrest.
3. No prosecution, suit or other legal proceeding shall be instituted, except with the
previous sanction of the State Government.

Select the correct answer by using the code given below


1.
2.
3.
4.

1 and 2
2 and 3
Only 2
Only 3

Solution- 3
J&Ks is AFSPA, 1990. In Tripura it is revoked. Last statement- It is Central Government

211. Identify the place based on given statements


1. It is on the bank of river Thames
2. It is notable for its association with the recent sealing of Magna Carta by King John
Select the correct option
1.
2.
3.
4.

Longmede
Abingdon
Windsor
Runnymede

Solution- 4
http://www.theguardian.com/law/2015/jun/15/david-cameron-british-bill-of-rightssafeguard-legacy-magna-carta

212. Consider the statements


Iasbaba.com

Page 126

IASbaba.com
1.
2.
3.
4.

Its people include Nagas, Kuki and Meeties


It is bounded by Myanmar to its East
Its Kingdom was one of the Princely States of British India
Famous for Swamp Deer

Select the correct option


1.
2.
3.
4.

Nagaland
Mizorum
Manipur
Meghalaya

Solution- 3

213. Consider the following


1.
2.
3.
4.
5.
6.

Nathu La
Bum La
Sipki La
Pensi La
Chunsul La
Lipulekh Pass

Which of the above are Border Personnel Meeting Points of India-China?


1.
2.
3.
4.

1, 2, 3, 4
2, 3, 4, 5 and 6
1, 2, 5 and 6
1, 3, 4 and 5

Solution- 3
http://economictimes.indiatimes.com/news/defence/india-china-to-open-borderpersonnel-meeting-point-along-line-of-actual-control/articleshow/48268389.cms

Iasbaba.com

Page 127

IASbaba.com
https://en.wikipedia.org/wiki/Nathu_La

214. Identify the place based on given information


1.
2.
3.
4.

It is an Oasis
The architecture is a mix of Roman-Graeco Techniques with influence of Persian
It is UNESCO designated World Heritage Site
Recently destroyed by ISIS

Select the correct option


1.
2.
3.
4.

Damascus
Syria
Palmyra
Samarra

Solution- 3
http://www.theguardian.com/world/2015/jun/23/isis-destroys-palmyra-shrines-in-syria

215. Consider the following statements


1. It is famous for Terracotta Warriors
2. It is located at northern foot of Lishan Mountain
3. It is a World Heritage Site
Select the correct option
1.
2.
3.
4.

Xian
Xianyang
Shaanxi Province
Both 1 and 3

Solution- 4
http://whc.unesco.org/en/list/441

Iasbaba.com

Page 128

IASbaba.com
216. Consider the following statements
1. It has Turkey in its East
2. Mediterranean Sea is at the South
3. Bulgaria is the North
Select the correct option
1.
2.
3.
4.

Italy
Serbia
Greece
Montenegro

Solution- 3

217. Which of the following are correctly matched?

Multipurpose Project
1.
2.
3.
4.

Pancheshwar Project
Bargi Project
Matatilla Project
Hasdeo Bango Project

River
i. Sharda River
ii. Narmada River
iii. Betwa River
iv. Mahanadi

Select the correct code/s


1.
2.
3.
4.

1, 2, 3 and 4
1, 3 and 4
2 and 3
1 and 4

Iasbaba.com

Page 129

IASbaba.com
Solution- 1
All are correct

218. One Belt, One Road Initiative of China will pass through which of the following?
1.
2.
3.
4.
5.

Black Sea
Mediterranean Sea
Caspian Sea
Red Sea
Yellow Sea

Select the correct code/s


1.
2.
3.
4.

1, 2, 3, 4 and 5
2, 3, 4 and 5
2, 3 and 4
1, 2 and 4

Solution- 4
One Belt, One Road includes- New Silk Road plus Maritime Silk Route
New Silk Route- Near Istanbul, it will cross Black Sea
Maritime Silk Route- Stretches till Greece through Red Sea and Mediterranean Sea
See this Map and try to make more questions around this. Like places and some symmetry
around the geography of regions associated with it. We will also post more questions

Iasbaba.com

Page 130

IASbaba.com

219. Consider the statements


1. It is Indias first Dinosaur Site
2. This Jurassic period site houses the fossils of flora

3. 15 km from Gadchiroli, near Maharashtras border with Andhra Pradesh.


Correct option

1. Wadadham Fossil Park


2. Salkhan Fossil Park
3. Ghughua Fossil Park
Iasbaba.com

Page 131

IASbaba.com
4. Indroda Fossil Park
Solution- 1
http://indianexpress.com/tag/wadadham-fossil-park/

220. India is second largest in production of Fishery in the world. Consider the statement
in this regard

1.
2.
3.
4.

India has utilized more than 70% of its fisheries potential


Gujarat is the leading producer in India followed by Andhra Pradesh
Inland fish production is more than marine fish production
Both Coastal and Deep Sea Fishery falls under State Government

Select the WRONG option


1.
2.
3.
4.

1 and 3
2 and 4
Only 1
1, 2 and 4

Solution- 4
Indias fishery potential is hugely untapped. AP is the leading producer. Coastal fishery is
under State but Deep Sea Fishing is governed by UNION due to Scientific Research and
jurisdiction under EEZ.

221. Which of the following animals have suicidal tendencies or self-destruction or


autothysis?
1.
2.
3.
4.
5.
6.

Carpenter ant
Bat
Pea aphid
cat
Termites
Dogs

Select the correct code


Iasbaba.com

Page 132

IASbaba.com
1.
2.
3.
4.

1, 2, 3, 4, 5 and 6
2, 3, 4 and 6
2 and 5
1, 3, 5 and 6

Solution- 4
Read about Autothysis from Google

222. Consider the following statements


1. It has four World Heritage Sites next highest to Russia in Central Asia
2. Samarkand is one the World Heritage Site
3. It has Caspian Sea to the West and Afghanistan to the South
Select the correct code
1.
2.
3.
4.

Kazakhstan
Uzbekistan
Tajikistan
Turkmenistan

Solution- 2
Samarkand is in Uzbekistan [Those who knew this, will easily solve]

223. Consider the following statements


1. Arab State of Persian Gulf consists of seven states namely Iraq, Kuwait, Saudi Arab,
Bahrain, Qatar, UAE and Oman
2. All are part of Gulf Cooperation Council (GCC)
3. India has Free Trade Agreement (FTA) with GCC country, UAE.
Select the correct code
1. 1, 2, 3
2. 1 and 2
3. Only 1
Iasbaba.com

Page 133

IASbaba.com
4. None
Solution- 3
First statement is correct but in second statement, GCC excludes Iraq. India is under
negotiation with GCC. Two rounds have been done but no ratification yet.

224. Consider the following statements


1. It is one the World Heritage Sites and an Ancient University of Indian Subcontinent
2. It was a major centre of learning for Buddhism, Jainism and Hinduism.
3. It became a renowned centre of Buddhist religion and culture during the royal
Patronage of Pala Dynasty.
4. It was established by Dharmapala of Pala Dynasty
Select the correct code
1.
2.
3.
4.

Nalanda
Takshashila
Vikramshila
Somapura

Solution- 4
Somapura Mahavihara was established by Dharmapala of Pala dynasty during late 8th
century in Bengal (Now in Bangladesh) and flourished for 400 years till 12th century. The
University spread over 27 acres of land of which the main complex was 21 acres was one of
the largest of its kind. It was a major center of learning for Bauddha Dharma (Buddhism),
Jina Dharma (Jainism) and Sanatana Dharma (Hinduism). Even today one can
find ornamental terracotta on its outer walls depicting the influence of these three
traditions.
http://whc.unesco.org/en/list/322
Iasbaba.com

Page 134

IASbaba.com

225. Consider the following


1.
2.
3.
4.

Royal Hill of Ambohimanga


Rock Shelters of Bhimbetka
The Persian Garden
Osun-Osogbo Sacred Grove

What is the relation between above regions?


1.
2.
3.
4.

They are newly inscribed World Heritage Sites


They are Cultural Landscapes of World Heritage
They are nominated members of World Heritage
They are World heritage in danger

Solution- 2
http://whc.unesco.org/en/culturallandscape/#1

226. Consider the following regarding National Mission for Clean Ganga (NMCG)
1. The GoI has availed loan from International Development Bank and International
Bank for Reconstruction and Development (IBRD) to facilitate this project.
2. The Secretary to the Government of India, Ministry of Water Resources, River
Development and Ganga Rejuvenation (MoWR, RD & GR) is the current Chairman of
the Government Council of NMCG.

3. NMCG has been assigned the implementation of the World Bank assisted project of
NGRBA and had taken loan from World Bank.
Select the correct code

Iasbaba.com

Page 135

IASbaba.com
1.
2.
3.
4.

1, 2 and 3
2 and 3
1 and 2
Only 3

Solution- 1
India Year Book- Water Resources
http://envfor.nic.in/sites/default/files/nmcg-ad-05062014.pdf

227. Which of the following are associated with diabetes mellitus, a common disease in
adults?
1.
2.
3.
4.

Higher sugar level in blood


Lower sugar level in blood
Lower insulin level in blood
Higher insulin level in blood

Select the correct answer by using the codes given below


1.
2.
3.
4.

1 and 3
1 and 4
2 and 4
2, 3 and 4

Solution- 1
With diabetes mellitus, either your body doesn't make enough insulin or it can't use
the insulin it does produce or a combination of both. Means, there will be no excess of
insulin in either case.

228. Consider the following


Subtropical gyre

Name of the gyre

1.
2.
3.
4.
5.

Columbus
Majid
Turtle
Heyerdhal
Navigator

North Atlantic
South Atlantic
North Pacific
South Pacific
Indian Ocean

Which of the above are correctly matched?


Iasbaba.com

Page 136

IASbaba.com
1.
2.
3.
4.

1, 2, 3, 4 and 5
3, 4 and 5
1, 3 and 4
1, 2 and 5

Solution- 3
Option 2 and 5 are exchanged to baffle you

229. Consider the statements


1.
2.
3.
4.

Tropic of Cancer passes through this state


Myanmar in the East, Bangladesh in the West
Phawngpui Blue Mountain National Park
Dhaleshwari river flows through the state

Select the correct code


1.
2.
3.
4.

Mizoram
Meghalaya
Nagaland
Manipur

Solution- 1

230. Consider the statements


1. The 5000-year-old 626-hectare archaeological site of The Sacred City of Caral-Supe is
situated here
2. Recently a trio-mud statuette was discovered in the archaeological site.
3. It borders Columbia to the North and Chile to the South
Select the correct code
1. Bolivia
2. Ecuador
Iasbaba.com

Page 137

IASbaba.com
3. Peru
4. Brazil
Solution- 3
http://www.businessinsider.com/afp-3800-year-old-statuettes-found-in-peru-2015-6?IR=T

231. Consider the following


1.
2.
3.
4.

Shikhara
Mandapa
Vahan
Gopuram

Which of the above are common between Nagara and Dravida Temple style?
1.
2.
3.
4.

1, 2, 3 and 4
2 and 4
1, 2 and 3
1, 3 and 4

Solution- 3
Gopuram is part of Dravida not Nagara

232. Consider the following statements


1. Its a Mountain Range in Central Asia where China, Russia, Mongolia and Kazakhstan
come together.
2. Rivers like Irtysh and Ob have their headwaters here
3. It means, The Gold Mountain in Mongolian
Select the correct code
1. Pamir Mountains
Iasbaba.com

Page 138

IASbaba.com
2. Altai Mountains
3. Kunlun Mountains
4. Tian Shan Mountains

Solution- 2

233. Consider the following statements regarding El-Nino


1. El-Nino is a warm phase of El Nino Southern Oscillation (ENSO) leading to Droughts
and low rainfalls only.
2. Warming of Eastern Pacific due to El-Nino relates to droughts in India
3. Every El Nino results in a deficient monsoon season
Select the correct code/s
1.
2.
3.
4.

1 and 2
2 and 3
Only 2
None

Solution- 4
http://indianexpress.com/article/india/weather-india-2/thank-heavens-el-nino-unlikely-tohave-big-impact-on-monsoon/
http://www.business-standard.com/article/current-affairs/el-nino-and-its-likely-effect-onindia-115051800590_1.html

234. India has Eight Missions under National Action Plan for Climate Change. Consider the
statements regarding them
1. Two of the Missions seek to slow down the growth of Indias emissions.
2. Three of them are service missions and seek to create more knowledge on useful
climate responses.

Iasbaba.com

Page 139

IASbaba.com
3. Mission on Agriculture, Afforestation and Himalayan Eco-System are about initiating
measures to adapt to the effects of climate change
Select the WRONG codes
1.
2.
3.
4.

1, 2 and 3
1 and 3
Only 2
None

Solution- 1

Three of them on solar energy, afforestation and energy efficiency seek to slow down
the growth of Indias emissions. Another three on agriculture, water and Himalayan ecosystems are about initiating measures to adapt to the effects of climate change. The
remaining two on sustainable habitat and strategic knowledge are service missions
and seek to create more knowledge on useful climate responses.

235. Consider the following


1. The river traverses through three Indian States
2. The famous Hundru Fall is created on the course of it.
3. It is famous for its placer deposits.
Name the river
1.
2.
3.
4.

Brahmani
Damodar
Subarnarekha
Mahanadi

Solution- 3
Placer Deposit- Subarnarekha is famous for Gold. Hundru fall is also on its course. It traverse
through Jharkhand, Orissa and West Bengal.

Iasbaba.com

Page 140

IASbaba.com
http://timesofindia.indiatimes.com/india/GSI-forwards-Subarnarekha-gold-mining-reportto-Jharkhand/articleshow/48279862.cms

236. Consider the following


1.
2.
3.
4.
5.

Meghalaya
Mizoram
Nagaland
Manipur
Arunanchal Pradesh

Inner Line Permit is applicable to?


1.
2.
3.
4.

1, 2 and 3
2, 3 and 5
1, 2, 4 and 5
1, 2, 3 and 5

Solution- 2
http://www.mdoner.gov.in/content/inner-line-permit

237. European Union is a politico-economic Union of 28 member states. Euro is thought be


the common currency of its members but many members do not use Euro. Consider the
following
1.
2.
3.
4.
5.

Greece
Poland
UK
Luxemburg
Bulgaria

Select the nations of EU not using EURO as their official currency?


1.
2.
3.
4.

1, 3, 4 and 5
2, 3 and 5
1, 2 and 3
1, 2, 3 and 5

Iasbaba.com

Page 141

IASbaba.com
Solution- 2
Greece and Luxemburg uses EURO.
https://www.ecb.europa.eu/euro/intro/html/map.en.html

238. Consider the following

1.
2.
3.
4.

Galathea National Park


Mouling National Park
Nameri National Park
Harike Wetland

Punjab
Assam
Arunanchal Pradesh
Goa

Select the correct match


1.
2.
3.
4.

2, 3 and 4
2 and 3
1, 2, 3 and 4
None

Solution- 4
Galathea National Park

Andaman and Nicobar

Mouling National Park

Arunanchal Prades

Nameri National Park

Assam

Harike Wetland

Punjab

239. The branch of science dealing with the improvement of human race is
1.
2.
3.
4.

Eugenics
Genealogy
Cloning
Euphenics

Iasbaba.com

Page 142

IASbaba.com
Solution- 1

240. Department of Atomic Energy comes under


1.
2.
3.
4.

Home Ministry
Prime Ministers Office
Ministry of Science & Technology
Ministry of Power

Solution- 2

241. Consider the following statements w.r.t Lead (Pb), an element of controversy in
recent times
1. If entered in body, it is most likely to be treated as Iron (Fe)
2. Lead is poisonous but not radioactive
3. The low density and electrical conductivity of lead makes it very effective as
Radiation Shield.
Select the correct code
1.
2.
3.
4.

1, 2 and 3
1 and 3
1 and 2
None

Solution- 4

Your body treats lead like calcium, so it accumulates in places where calcium is stored, such
as your bones. Because calcium has essential roles in hormones, muscle contractions and
the communication between nerve cells, lead can interfere with these processes. Lead is
radioactive also though with lower half life.

Iasbaba.com

Page 143

IASbaba.com
Lead shielding refers to the use of lead as a form of radiation protection to shield people or
objects from radiation so as to reduce the effective dose. But it has nothing to do with
electrical conductivity. High Density of lead makes it a better radiation shield. Because of
leads high density and large number of electrons, it is well suited to scattering x-rays and
gamma-rays

242. Consider the following


1. The density of Earth is less than Moon
2. The double planet or condensation hypothesis claimed that composition of Moon
and Earth are different.
3. If the moon passes through the part of the Earths shadow called the umbra, a total
lunar eclipse occur
Select the correct code
1.
2.
3.
4.

1 and 2
2 and 3
Only 1
Only 3

Solution- 3
By examining rocks brought back from the lunar surface, geologists found that the density of
the moon (3.3 g/cm3) is less than the density of Earth (5.5 g/cm3). This indicates that the
moon doesn't have an iron core.
The double planet, or condensation hypothesis, claimed that the moon: The double planet
hypothesis of the 1960s claimed that the moon and Earth had formed at about the same
time. Upon studying lunar samples, however, scientists found that the composition of the
moon was different from the composition of Earth. If the double planet hypothesis was
correct, then the composition of both bodies should've been the same, and it wasn't.

When the moon passes through the umbra, a total lunar eclipse occurs. In a lunar eclipse,
the moon passes through the Earths shadow, created by the sun, which causes its light to
dim

Iasbaba.com

Page 144

IASbaba.com
243. Consider the following
1. If a rock is taken from Earth to Moon, its mass will change.
2. Gravitational pull of Moon is stronger than Earth
3. The Moon orbits Earth in prograde direction and completes one rotation in
approximately in 23 Days
Select the correct code
1.
2.
3.
4.

Only 1
1 and 3
1, 2 and 3
None

Solution- 4
No change in mass. Weight will change. Gravitational pull of Earth is more. 27.6 days for
rotation not 23

244. Consider the following


1.
2.
3.
4.

Banana
Foods rich in Potassium
CT Scan
Brazil Nuts

Which of the above have tendency to expose you for radioactive radiations?
1.
2.
3.
4.

1, 2, 3 and 4
2 and 3
2, 3 and 4
Only 3

Solution- 1
CT Scan Everyone knows about it

Iasbaba.com

Page 145

IASbaba.com
Banana- the radiation exposure from consuming a banana is approximately 1% of the
average daily exposure to radiation
Besides bananas, other foods that are rich in potassium (and therefore in 40K)
are potatoes, kidney beans, sunflower seeds, and nuts. Brazil nuts in particular are not only
rich in 40K but may also contain significant amounts of radium, which have been measured
at up to 444 Bq/kg.

245. Consider the following statements


1. The colour of blood plasma is due to presence of Iron (red in colour)in our body
2. RBC and Erythrocytes are common types of blood cells present in haemoglobin and
lungs respectively
3. Due to the phenomenon of osmosis water enters into RBCs
Select the correct code
1.
2.
3.
4.

1, 2 and 3
Only 3
1 and 3
None

Solution- 2
The color of erythrocytes is due to the heme group of hemoglobin. The blood plasma alone
is straw-colored, but the red blood cells change color depending on the state of the
hemoglobin: when combined with oxygen the resulting oxyhemoglobin is scarlet, and when
oxygen has been released the resulting deoxyhemoglobin is of a dark red burgundy color.
RBC is also known as Erythrocytes. Third statement is right.

246. Consider the following


1. Alpha Particle
Iasbaba.com

Page 146

IASbaba.com
2. An Electron
3. Proton
Arrange them in increasing order of wavelength if Kinetic Energy of all of them is same?
1.
2.
3.
4.

1-3-2
2-1-3
3-2-1
2-3-1

Solution- 1

Kinetic energy is given by E= (1/2)mv^2.


Same kinetic energy means electron has highest velocity (to make for its small mass)
Assume mass of electron as 1 unit. Mass of proton approx 1836 and mass of alpha particle is
4 times i.e is 7344.
Now calculate velocity in each case.
For electron v= square root (2E)
For proton v = square root (2E/1836)
For alpha particle v=square root (2E/7344)

Also de-Broglie wavelength is given by Lambda= h/mv

Now calculate momentum in each case


For electron mv = square root (2E)*1
For proton v = square root (2E/1836)*1836
For alpha particle v=square root (2E/7344)*7344

Clearly electron has least momentum. Hence it has largest de-Broglie wavelength. And
Alpha particle has least wavelength.

Iasbaba.com

Page 147

IASbaba.com
247. If Earth revolves around the Sun in circular path rather than elliptical path, what
would happen?
1.
2.
3.
4.

Difference between seasons will increase


The Earth would become very hot
Difference between seasons will be reduced
The Earth would become very cold

Solution- 3

248. Permanent hardness of water is due to the presence of


1.
2.
3.
4.

Bicarbonate of Calcium and Magnesium


Sulphates of Calcium and Magnesium
Sulphates of Sodium and Calcium
Carbonates of Sodium and Calcium

Solution- 2

249. Consider the following


1.
2.
3.
4.
5.

Vacuoles
Cell membranes
Chloroplast
Cytoplasm
Nucleus

Which of them are common to both, Animal and Plant Cell?


1.
2.
3.
4.

1, 2, 3 and 4
1, 2, 4 and 5
2, 3, 4 and 5
2, 4 and 5

Solution- 2
Plant Cell has Chloroplast unlike Animal Cell.

Iasbaba.com

Page 148

IASbaba.com
250. Consider the following
1. Light and Sound Waves are Electromagnetic Waves
2. Both shows the property of Diffraction, Reflection and Refraction
3. While Sound is Transverse waves, light is Longitudinal in nature
Select the correct code
1.
2.
3.
4.

1, 2and 3
1 and 2
Only 2
Only 3

Solution- 3
Sound is Mechanical Wave not electromagnetic. Sound is Longitudinal and light is
Transverse. Second statement is true

251. Consider the following


1. Zinc Deposits
2. Gold Deposits
3. Uranium Deposits

Karnataka
Rajasthan
Andhra Pradesh

Which of them is/are wrongly matched?


1.
2.
3.
4.

1, 2 and 3
1 and 2
Only 2
Only 3

Solution- 2
Zinc Deposits- Rajasthan and Gold Deposits in Karanataka

252. Consider the following about JAM Trinity introduced by GoI


1. JAM trinity will reduce the dependency on Mobile services
2. It will give more penetration to Post Offices
3. It will cut down the leakages in subsidy
Iasbaba.com

Page 149

IASbaba.com
Select the correct code
1.
2.
3.
4.

1, 2 and 3
1 and 3
2 and 3
Only 3

Solution- 3
JAM Trinity is Jan Dhan Yojana, Aadhar and Mobile linkage. It will increase the mobile
penetration. Rest statements are self explanatory

253. Consider the following


1. Ferns
2. Mosses
3. Blue green algae
Which of the above can be used as Bio-Fertilizers?
1.
2.
3.
4.

1, 2 and 3
1 and 2
2 and 3
Only 3

Solution- 1

254. Consider the statements about bio-fertilizers


1. They increases the yield of the crop
2. They can produce vitamins and growth promoting biochemical
3. They replaces other fertilizers when put in soil
Iasbaba.com

Page 150

IASbaba.com
Select the correct answer using the code
1.
2.
3.
4.

1 and 2
1 and 3
Only 1
1, 2 and 3

Solution- 1
They complement other fertilizers and never replace them.

255. Consider the following


1.
2.
3.
4.
5.

Hungary
Bosnia
Austria
Albania
Croatia

Which of the above has/have Adriatic Sea as a boundary?


1.
2.
3.
4.

1, 3, 4 and 5
2, 3 and 4
1, 2 and 3
2, 4 and 5

Solution- 4
Check the Map

256. Consider the following


1.
2.
3.
4.

Troposphere
Stratosphere
Mesosphere
Thermosphere

Iasbaba.com

Page 151

IASbaba.com
Which of the following is/are responsible for the deflection of radio-waves?
1.
2.
3.
4.

Only 4
2 and 4
2 and 4
None

Solution- 1
Its Ionosphere- Extension of Thermosphere

257. Consider the following


1.
2.
3.
4.

Bug
Scorpion
Lobster
Mite

Which of the above have equal pairs of legs?


1.
2.
3.
4.

1, 2 and 3
2 and 4
1, 2 and 3
3 and 4

Solution- 2
Bug- 3 pairs
Scorpion- 4 pairs
Mite- 4 pairs
Lobster- 5 pairs

258. Consider the following


1.
2.
3.
4.
5.

Lymphatic Filariasis
Leprosy
TB
Rabies
Malaria

Iasbaba.com

Page 152

IASbaba.com
6. Dengue
7. Snakebite
Which of the above are Neglected Tropical Disease classified under WHO?
1.
2.
3.
4.

1, 2, 4, 6 and 7
2, 3, 5 and 6
1, 4, 5, 6 and 7
2, 3, 4 and 5

Solution- 1
There are 17 NTD under WHO. Try to remember them as they are time and again in news.

259. Consider the following statements


1.
2.
3.
4.

Tissue culture can be done in plants as well as animals


It increases the genetic diversity
It provides a great scope for combating hunger
The susceptibility of produced species are decreased towards environment and pests

Select the correct answer from the above codes


1.
2.
3.
4.

1, 2 and 3
1, 3 and 4
2 and 4
1 and 3

Solution- 4
Yes it can be done in plants as well as animals. But in plants, as all the plants are genetically
similar, there is reduction is genetic diversity. As genetic profile will remain same so
susceptibility towards environment and pests will not decrease. It will be equal to parent
species. More and more crops can be created from one plant so great scope for combating
hunger.

260. It describes a genetically distinct variety, population or race within a species, which is
adapted to a specific environment1.
2.
3.
4.

Phenotype
Ecophenotype
Ecotype
Genotype

Iasbaba.com

Page 153

IASbaba.com
Solution- 3

261. The increasing amount of CO2 in the air is slowly raising the temperature of
atmosphere, because it absorbs
1.
2.
3.
4.

All solar radiation


UV part of radiation
The water vapour of the air and raises its heat
The infra-red part of solar radiation

Solution- 4

262. Consider the following


1.
2.
3.
4.
5.

Methane
Oxygen
Carbon dioxide
Hydrogen
Acetylene

Mixture of which of the above gases are responsible for most of the explosions in mines?
1.
2.
3.
4.

1, 2 and 5
2, 3 and 4
1 and 2
2, 3 and 4

Solution- 3

263. Consider the following


1.
2.
3.
4.

Tiger Moth
Dolphin
Mice
Grasshopper

Iasbaba.com

Page 154

IASbaba.com
5. Frog
6. Bats
Which of the above can produce Ultrasonic sounds?
1.
2.
3.
4.

1, 2, 3, 4, 5 and 6
2, 4 and 6
2 and 6
1, 2 and 6

Solution- 1

264. Animal Husbandry and Plant breeding programmes are examples of


1.
2.
3.
4.

Artificial Selection
Reverse Evolution
Natural Selection
Mutation

Solution- 1

265. Plants require Micro-nutrients for their growth and development. Consider the
following w.r.t it
1.
2.
3.
4.
5.
6.
7.

Boron
Calcium
Iron
Magnesium
Manganese
Potassium
Sodium

Select the correct answer using the codes


1. 2, 4, 6 and 7
2. 1, 3, 5 and 7
3. 1, 2, 3, 5 and 6
Iasbaba.com

Page 155

IASbaba.com
4. 1, 2, 3, 4, 5, 6 and 7

Solution- 2
Calcium, Magnesium, Potassium are macro-nutrients.

266. Consider the statements


1. Bay of Bengal in the South
2. Situated on Shan plateau
3. One of the most important river systems is Salween
Select the correct answer from the code below
1.
2.
3.
4.

Thailand
Bangladesh
Myanmar
Vietnam

Solution- 3
267. Arrange the rivers from North to South
1.
2.
3.
4.

Kishenganga
Penganga
Ganga
Wainganga

Select the correct answer from code


1.
2.
3.
4.

1-2-3-4
3-1-2-4
1-3-4-2
1-3-2-4

Solution- 3

268. Recently PM of Canada handed over 900 years old Khajuraho Temples Sculpture.
What is the name of it?
1. Dancing Lady
2. Parrot Lady
Iasbaba.com

Page 156

IASbaba.com
3. Dancing Parrot
4. Dancing Queen
Solution- 2

269. Consider the statements


1.
2.
3.
4.

Cold water corals are found in North-East as well as North-West Atlantic Ocean
Cold water corals also have symbiotic relation with algae like tropical corals
Cold water corals have small polyps as compared to tropical corals
Worlds largest known cold water coral is Rost Reef located in Norway

Select the correct answer using the codes


1.
2.
3.
4.

1, 2 and 3
2 and 4
2 and 3
1 and 4

Solution- 4
Cold water or Deep water corals do not have symbiotic relation with algae unlike tropical
corals. The polyps of cold water corals are bigger than tropical corals
270. Consider the following
1.
2.
3.
4.
5.

Prithvi- Surface to Surface


BrahMos- Air to Air
Astra- Air to Air
Pragati- Surface to Surface
Akash- Air to Surface

Which of the above are correctly matched?


1.
2.
3.
4.

1, 3, 4 and 5
1, 2 and 4
1, 2, 3 and 4
2, 3 and 4

Solution- 3
BrahMos- All variants

Iasbaba.com

Page 157

IASbaba.com
Akash- Surface to Air

271. HFCs are potent green house gas. It finds applications in various fields like
1.
2.
3.
4.
5.

As refrigerant
Treatment for Asthma
Agents for plastic thermal insulation foams
As propellant
Fire-fighting agents

Select the correct answer from the codes given


1.
2.
3.
4.

1, 2, 3, 4 and 5
1, 3, 4 and 5
1, 3 and 5
1, 2 and 4

Solution- 1
http://www.britannica.com/science/hydrofluorocarbon
272. Consider the following
1.
2.
3.
4.
5.
6.

Shark
Termites
Crow
Racoons
Vulture
Ants

Which of the above are scavengers?


1.
2.
3.
4.

1, 4 and 5
2, 3, 4 and 5
1, 2, 4, 5 and 6
1, 2, 3, 4, 5 and 6

Solution- 4
All are scavengers

Iasbaba.com

Page 158

IASbaba.com
273. Consider the statements
1. Uniform Civil Code is mentioned under Article 46, Directive Principle of States Policy.
2. Indian States are still to come to its adoption
Chose the correct statement/s
1.
2.
3.
4.

Only 1
Only 2
Both
None

Answer- 4
UCC is mentioned under Article 44 not 46. GOA is the only state to have UCC, so statement 2
is also incorrect

274. Consider the following


1.
2.
3.
4.

Parliament
States Legislatures
Supreme Court
Union Territory Legislature

STATUTES are enacted by


1.
2.
3.
4.

1, 2 and 3
1, 3 and 4
1, 2 and 4
1, 2, 3 and 4

Answer- 3
Statutes are enacted by Parliament, State Legislatures and Union Territory Legislatures.

Iasbaba.com

Page 159

IASbaba.com
275. Consider the following statements regarding Jurisdiction of Supreme Court
1. The appellate jurisdiction of the Supreme Court can be invoked by a certificate
granted by the President of India concerned under Article 134 of the Constitution in
both civil and criminal cases.
2. The Supreme Court has special advisory jurisdiction in matters which may specifically
be referred to it by the President of India under Article 143 of the Indian
Constitution.
3. The dispute Indian and Bangladesh on Water Treaty comes under Original
Jurisdiction of Supreme Court of India
Select the WRONG statement/s
1.
2.
3.
4.

1 and 2
2 and 3
1 and 3
1, 2 and 3

Answer- 3
Statement 1- Under Article 134 of the Constitution of India in criminal cases, an appeal shall
lie to the Supreme Court from the decision of a High Court, if the High Court. So statement
should beThe appellate jurisdiction of the Supreme Court can be invoked by a certificate granted by
the High Court concerned under Article 134 of the Constitution in both civil and criminal
cases.
Statement 3 is wrong

276. Consider the following


1.
2.
3.
4.
5.

Fire Service
Home Guards
CISF
Police
Civil Defence

Which of the above comes under Law Enforcement Agencies of India?


Iasbaba.com

Page 160

IASbaba.com
1.
2.
3.
4.

1, 2, 3 and 5
2, 4 and 5
2, 3, 4 and 5
1, 2, 3, 4 and 5

Answer- 4

277. Consider the following countries


1.
2.
3.
4.
5.

Algeria
Mexico
Libya
Egypt
Mali

Arrange the countries (West to East) through which Tropic of cancer passes.
1.
2.
3.
4.

2-5-1-3-4
5-1-2-4-3
3-1-4-2-5
2-1-5-4-3

Answer- 1

278. Consider the statements


1. It is the smallest and rarest wild pig
2. It is a schedule-I species under Wild Life Protection Act, 1972
3. Mostly found in Manas Tiger Reserves and listed under Critically Endangered Species
Select the correct answer
1.
2.
3.
4.

Pangolin
Wild Boar
Pygmy Hog
Forest Hog

Answer- 3

Iasbaba.com

Page 161

IASbaba.com
http://www.thehindu.com/sci-tech/energy-and-environment/21-pygmy-hog-nests-foundin-manas-national-park/article5822945.ece

279. Consider the following


1.
2.
3.
4.
5.
6.

Dugong
Hispid Hare
Terrapin
Logger Head Turtle
Lynx
Cheer Pheasant

Which of the above are MAMMALS listed under Wildlife Protection Act, 1972?
1.
2.
3.
4.

1, 2, 3, 4, 5 and 6
2, 4 and 6
2, 3 and 5
1, 2 and 5

Answer- 4
Terrapin and Logger Head Turtle are listed under Amphibians and Reptiles while Cheer
Pheasant is a Bird

280. Consider the following statements


1. Traditional Knowledge Digital Library (TKDL) is a collaborative initiative of Ministry of
Health and Family Welfare, DRDO and CSIR
2. Traditional Knowledge is mentioned under Convention of Biodiversity.
3. Nagoya Protocol and Cartagena Protocol also involve Traditional Knowledge
Select the correct statement/s
1.
2.
3.
4.

1, 2 and 3
2 and 3
1 and 2
Only 2

Answer- 4

Iasbaba.com

Page 162

IASbaba.com
Collaboration- The Traditional Knowledge Digital Library (TKDL) is an Indian digital
knowledge repository of the traditional knowledge, especially about medicinal plants and
formulations used in Indian systems of medicine. Set up in 2001, as collaboration between
the Council of Scientific and Industrial Research (CSIR) and the Department of Ayurveda,
Yoga and Naturopathy, Unani, Siddha and Homoeopathy (Dept. of AYUSH), Ministry of
Health & Family Welfare, Government of India.
Convention of Biodiversity- Access and Benefit Sharing- Nagoya not Cartagena

281. Consider the following


Below is the list of indigenous species and its location
1.
2.
3.
4.

Quinoa
Kava
Hoodia
Ayahuasca

Amazon Basic
Western Pacific
Namib Desert
Andes

Which of the above is/are correctly matched?


1.
2.
3.
4.

1, 2, 3 and 4
1 and 2
2 and 3
3 and 4

Answer- 3
ExplanationQuinoa

Andes

Kava

Western Pacific

Hoodia

Namib Desert

Ayahuasca

Iasbaba.com

Amazon Basin

Page 163

IASbaba.com
282. Which of the following can be used to check whether drinking water contains Gamma
emitting isotope or not?
1.
2.
3.
4.

Scintillation Counter
Lead Plate
Spectrophotometer
Microscope

Answer- 1

283. Consider the following statements regarding newly launched Vanbandhu Kalyan
Yojana
1. It covers all tribal people and all areas with tribal population across the country
2. It focuses on improving the quality of education of Tribal people
3. One of the main objectives is to address the infrastructural gap with focus on quality
Select the correct answer using codes
1.
2.
3.
4.

1, 2 and 3
2 and 3
1 and 2
Only 1

Solution- 1
VKY a strategic process. It aims at creating enabling environment for need based and
outcome oriented holistic development of the tribal people.This process envisages to
ensure that all the intended benefits of goods and services under various
programmes/schemes of Central as well as State Governments actually reach the target
groups by convergence of resources through appropriate institut ional mechanism.
Scope: It covers all tribal people and all areas with tribal population across the country.
Objectives
1.
2.
3.
4.
5.

Improving the quality of life in tribal areas


Improving the quality of education
Qualitative and sustainable employment for tribal families
Bridging infrastructure gaps with focus on quality
Protection of tribal culture and heritage

Iasbaba.com

Page 164

IASbaba.com

284. The Living Planet Report is the world's leading, science-based analysis on the health
of our planet and the impact of human activity. It is released by
1.
2.
3.
4.

United Nation Environment Program (UNEP)


WWF
UNESCO
IUCN

Solution- 2

285. Match the following defence exercises of India


1.
2.
3.
4.

Yudh Abhyas
Nomadic Elephant
EKUVERIN
Exercise Malabar

India-USA
India-Mongolia
India-UK
India-USA

Select the correct match


1.
2.
3.
4.

1, 2, 3 and 4
2 and 3
1 and 2
1, 2 and 4

Solution- 4
EKUVERIN- India- Maldives
INDIA AND U.S
Malabar: It is Naval exercise takes place in India and U.S. alternatively
Yudh Abhyas: It is Army exercise takes place in India alternatively
Red flag: It is Air Force exercise takes place in only in Nevada Desert of U.S.
Cope: Its Army exercise in U.S

Iasbaba.com

Page 165

IASbaba.com
INDIA FRANCE: - VARUNA: Joint exercise in Air Force
INDIA AND U.K. - GARUDA: Air Force
INDIA AND MALDIVES- EKUVERIN: IN all forms but mostly in Army
INDIA AND MANGOLIA- NOMADIC ELEPHANT: Mostly in Army
INDIA AND CHINA- HAND IN HAND: In all forms
INDIA AND SRI LANKA- SLINEX: Joint exercise in Navy
INDIA AND SOUTH EAST ASIA (ESPECIALLY SINGAPORE)- SIMBEX: Joint exercise in Navy
INDIA AND THAILAND- Maitree: focus being on counter-insurgency operations in urban and
rural terrain
INDIA and RUSSIA- INDRA 2012 --- anti terrorism activity

286. Two pieces of conductor of same material and of equal length are connected in series
with a cell. One of the two pieces has cross-sectional area double that of the other.
Which one of the following statements is correct in this regard?
1.
2.
3.
4.

The thinner one will have strong current passing through it


The thicker one will have strong current pass through it
Both will have equal current but thinner one will have more heat
Both will have equal current but thicker one will have more heat

Answer- 3
287. Which one among the following is a strong smelling agent added to LPG cylinder to
help in the detection of gas leakage?
1.
2.
3.
4.

Thioethanol
Ethanol
Acetone
Methanol

Iasbaba.com

Page 166

IASbaba.com
Answer- 1

288. Gravitational force shares a common feature with electromagnetic force. In both
cases, the force is
1.
2.
3.
4.

Between massive and neutral objects


A short range
A long range
Between charged object

Answer- 3

289. Arrange the following hills of southern India from north to south
1.
2.
3.
4.

Nilgiri
Cardamom
Nallamala
Anaimalai

Select the correct answer using the code given below


1.
2.
3.
4.

3-1-4-2
2-3-4-1
3-2-4-1
1-2-3-4

Answer-1

290. Three important rivers of the Indian subcontinent have their source near the
Manasarovar Lake in the Great Himalayas. Which among the following are the rivers?
1.
2.
3.
4.

Indus, Sutlej and Beas


Brahmaputra, Yamuna and Indus
Indus, Sutlej and Brahmaputra
Sutlej, Jhelum and Yamuna

Iasbaba.com

Page 167

IASbaba.com

Answer- 3

291. Match the following correctly


List I (Island)

List II (Location)

1.
2.
3.
4.

Mauritius
Medagascar
Andaman-Nicobar
Maldives

Continental island
Coral island
Volcanic island
Mountains island islands

Match the correct answer


1.
2.
3.
4.

1, 2 and 4
2 and 4
1 and 2
None

Answer- 4
Mountains island islands- Andaman and Nicobar
Coral island- Maldives

292. An association between roots of higher plants and fungi is called


1.
2.
3.
4.

Fern
Lichens
Mycorrhizae
Blue Green Algae

Answer- 3

Iasbaba.com

Page 168

IASbaba.com

293. Consider the following


1. Plants of this group are diploid.
2. They are well adapted to extreme conditions.
3. They grow bearing sporophylls in compact structures called cones.
The group in reference is
1.
2.
3.
4.

Monocots
Dicots
Pteridophytes
Gymnosperms

Answer- 4

294. Which one of the following sets of animals share a four chambered heart?
1.
2.
3.
4.

Amphibian, Reptiles, Birds


Crocodiles, Birds, Mammals
Crocodiles, Lizards, Turtles
Lizards, Mammals, Birds

Answer- 2

295. Protein is an important substance found in every cell in the human body. It has many
functions including
1.
2.
3.
4.
5.

Repair and maintenance of body tissue


Can act as source of energy
Creation of hormones
Formation of antibodies
Transportation and storage of molecules in the body

Iasbaba.com

Page 169

IASbaba.com
Select the correct answer using code
1.
2.
3.
4.

1, 2 and 5
2 and 5
1, 2, 3 and 4
1, 2, 3, 4 and 5

Answer- 4

296. Match the element with its associated functions/roles and choose the correct option
among given below

1. Boron

i. splitting of H2O to liberate O2 during photosynthesis

2. Manganese

ii. needed for synthesis of auxins

3. Molybdenum

iii. component of nitrogenase

4. Zinc

iv. Pollen germination

5. Iron

v. component of ferredoxin

Chose the correct code


1.
2.
3.
4.

1-iv, 2-i, 3-iii, 4-ii and 5-v


1-iii, 2-iv, 3-i, 4-v and 5-ii
1-iv, 2-ii, 3-iv, 4-iii and 5-i
1-iv, 2-iii, 3-i, 4-ii and 5-v

Answer- 1

Iasbaba.com

Page 170

IASbaba.com

297. Consider the following


1.
2.
3.
4.
5.
6.

Space Technology
Telemetry
Magnetic levitation
Blood Banking
Surgery
X-Ray

Cryogenics find application in which of the above?


1.
2.
3.
4.

1, 2, 3 and 5
1, 3, 4, and 6
1, 4, 5 and 6
1, 3, 4 and 5

Answer- 4
Telemetry and X-ray No cryogenic application here

298. Consider the following


1.
2.
3.
4.

Soil
Nutrients
Amount of oxygen dissolved in Water
Non-Green Plants

Which of the above are abiotic components?


1.
2.
3.
4.

1, 3 and 4
1, 2 and 3
2, 3 and 4
1, 2, 3 and 4

Solution- 2
Non-Green Plants-Biotic

Iasbaba.com

Page 171

IASbaba.com
299. Consider the following
1.
2.
3.
4.

Mutualism
Amensalism
Commensalism
Parasitism

Which of the above consists of positive interaction?


1.
2.
3.
4.

1 and 3
2 and 4
1, 2 and 3
1, 3 and 4

Solution- 4
The question is just asking about positive interaction. It doesnt mean it is exclusive of
negative interaction. Be careful while reading the questions.
Mutualism- Positive, Positive interaction
Amensalims- Only negative since one is getting harmed and the other has no effect
Commensalism- Positive, Neutral relationship
Parasitism- Positive, Negative Interaction (Positive is there)

300. Consider the following statements


1. A species with large area requirements for which protection of the species offers
protection to other species that share the same habitat
2. Can be used to help select the locations of potential reserves
3. Find the minimum size of these conservation areas or reserves, and to determine the
composition, structure and processes of ecosystems
Based on the above statements, select the best suited answer
1.
2.
3.
4.

Keystone species
Indicator species
Umbrella species
Edge species

Iasbaba.com

Page 172

IASbaba.com
Solution- 3
Self explanatory- For more, read on google

301. Consider the following


1.
2.
3.
4.
5.

Mosses
Mollusca
Lichens
Stoneflies
Greasewoods

Which of the above is/are indicator species?


1.
2.
3.
4.

1, 3 ad 5
2, 4 and 5
1, 2, 3 and 4
1, 2, 3, 4 and 5

Solution- 4
https://en.wikipedia.org/wiki/Indicator_species
http://www.britannica.com/science/indicator-species

302. Consider the statements regarding biodiversity


1. Marine diversity tends to be highest along coasts in Eastern Pacific.
2. Gamma diversity refers to diversity within a particular area, community or
ecosystem.
3. Alpha diversity is a measure of the overall diversity for different ecosystems within a
region
Select the correct statement/s
1.
2.
3.
4.

1, 2 and 3
2 and 3
Only 3
None

Solution- 4

Iasbaba.com

Page 173

IASbaba.com
Marine diversity tends to be highest along coasts in Western Pacific. Last two definitions are
interchanged.
303. Consider the following
1.
2.
3.
4.
5.
6.
7.

Intertidal Zone
Littoral Zone
Continental shelf
Kelp Forest
Coral Reefs
Pelagic zone
Seamounts

Arrange these marine habitats in correct order of occurrence


1.
2.
3.
4.

1, 2, 7, 4, 5, 6 and 3
2, 3, 1, 4, 7, 5 and 6
2, 1, 4, 5, 3, 6 and 7
3, 1, 6, 5, 4, 7 and 2

Solution- 3
Order- Littoral zone, Intertidal zone, Estuaries, Kelp forests, Coral reefs, Ocean banks,
Continental shelf, Neritic zone, Straits, Pelagic zone, Oceanic zone, Seamounts,
Hydrothermal vents, Cold seeps, Demersal zone and Benthic zone

304. Consider the following


1. Ten of the eighteen biosphere reserves of India are part of the World Network of
Biosphere Reserves, based on UNECSO Man and the Biosphere Program
2. Panchmarhi and Nokrek Biosphere Reserves are two of them part of World Network
of Biosphere Reserves
Select the correct answer
1.
2.
3.
4.

Only 1
Only 2
Both
None

Iasbaba.com

Page 174

IASbaba.com

Solution- 2
Nine of the eighteen biosphere reserves are a part of the World Network of Biosphere
Reserves, based on the UNESCO Man and the Biosphere (MAB) Programme list.
Name

States

Year

Nilgiri Biosphere Reserve

Tamil Nadu, Kerala, Karnataka

2000

Gulf of Mannar Biosphere Reserve

Tamil Nadu

2001

Sundarbans Biosphere Reserve

West Bengal

2001

Nanda Devi Biosphere Reserve

Uttarakhand

2004

Nokrek Biosphere Reserve

Meghalaya

2009

Pachmarhi Biosphere Reserve

Madhya Pradesh

2009

Simlipal Biosphere Reserve

Odisha

2009

Achanakmar-Amarkantak Biosphere Reserve Chhattisgarh, Madhya Pradesh 2012


Great Nicobar

Iasbaba.com

Andaman and Nicobar Islands

2013

Page 175

IASbaba.com

305. Match the following


IUCN Protected Area Management
1.
2.
3.
4.
5.

Landscape
National Park
Area with sustainable use of natural resources
Habitat
Wilderness area

Category
i.
ii.
iii.
iv.
v.

I
II
III
IV
V

Which is/are correctly matched?


1.
2.
3.
4.

1, 2 and 3
2 and 4
1, 3 and 5
1, 2, 3, 4 and 5

Solution-2
IUCN Protected Area Management Categories:
Category Ia Strict Nature Reserve
Category Ib Wilderness Area
Category II National Park
Category III Natural Monument or Feature
Category IV Habitat/Species Management Area
Category V Protected Landscape/Seascape
Category VI Protected Area with sustainable use of natural resources

Iasbaba.com

Page 176

IASbaba.com

306. Recently hydroponics have gained momentum due to its amazing properties and
utility. Consider the following regarding this
1.
2.
3.
4.

It uses water and soil


It uses nutrients without soil
It can increase food production
Has been used in Space Programs

Select the correct answer


1.
2.
3.
4.

1, 2 and 4
2, 3 and 4
1 and 2
2 and 3

Solution- 2
Hydroponics is a term applied to the cultivation of plants in nutrient solutions without the
use of soil. It uses nutrient solutions without soil. It has potential for future techniques of
food production when water availability will be less. It is already gaining momentum in
many countries. NASA is using this for its future space programmes like Mars inhabitation
etc.
307. Consider the following
1.
2.
3.
4.
5.
6.

Thorium
Silicon dioxide
Arsenic
Uranium
Potassium
Mercury

Contaminants of Fly Ash includes


1.
2.
3.
4.

2, 3 and 6
2, 3, and 4
2, 3, 4 and 5
1, 2, 3, 4 and 6

Solution- 4

Iasbaba.com

Page 177

Вам также может понравиться